Lower Extremity Flashcards

1
Q

A 17-year-old boy undergoes resection of the right distal femur to treat osteosarcoma. A large allogeneic corticocancellous bone graft is used for skeletal reconstruction of the 10-cm bony defect. An intramedullary vascularized fibular free flap is also placed in the construct. Which of the following is the most likely purpose of this additional procedure?
A) Decrease the immunogenicity of the allograft
B) Decrease the rate of infection
C) Increase creeping substitution in the allograft
D) Increase the immediate strength of the construct
E) Shorten union time

A

E) Shorten union time

The most likely reason to place an intramedullary fibular free flap in the allogeneic bone graft is to shorten union time. Free fibular flaps contain an intrinsic blood supply and osteogenic cells. These offer an alternative to the allograft and the capability of osteogenesis through osteoinduction. Thus, the biologic advantage of fibular free flaps is a shortened union time compared with the allograft, which solely depends upon creeping substitution for healing. Union times may be as short as 3 to 5 months. The healing of massive allografts is generally slow, superficial, and incomplete. The union times for allograft healing have been reported at 14 to 23 months for intercalary defects.

The fibula may add some strength to the overall construct but this is not the primary reason for its use in the scenario described. It also does not mitigate any potential antigenicity that the graft may have.

How well did you know this?
1
Not at all
2
3
4
5
Perfectly
2
Q

Most common reason for placing an intramedullary fibular free flap in an allogenic bone graft?
Because?

A

The most likely reason to place an intramedullary fibular free flap in the allogeneic bone graft is to shorten union time. Free fibular flaps contain an intrinsic blood supply and osteogenic cells.

How well did you know this?
1
Not at all
2
3
4
5
Perfectly
3
Q

Intramedullary fibular free flaps: Type/capability of osteogenesis?

A

Free fibular flaps contain an intrinsic blood supply and osteogenic cells. Free fibular flaps offer an alternative to the allograft and the capability of osteogenesis through osteoinduction.

The graft heals by creeping substitution and as such is only osteoconductive, unlike vascularized bone.

How well did you know this?
1
Not at all
2
3
4
5
Perfectly
4
Q

Biologic advantage of fibular free flaps compared with allograft

A

Free fibular flaps offer an alternative to the allograft and the capability of osteogenesis through osteoinduction. Thus, the biologic advantage of fibular free flaps is a shortened union time compared with the allograft, which solely depends upon creeping substitution for healing.

How well did you know this?
1
Not at all
2
3
4
5
Perfectly
5
Q

Union time of free fibula flap + allograft for lower extremity defect

A

Union times may be as short as 3 to 5 months. The healing of massive allografts is generally slow, superficial, and incomplete.

How well did you know this?
1
Not at all
2
3
4
5
Perfectly
6
Q

The union times for allograft healing have been reported at _______ for intercalary defects

A

The union times for allograft healing have been reported at 14 to 23 months for intercalary defects.

How well did you know this?
1
Not at all
2
3
4
5
Perfectly
7
Q

Nonvascularized massive allografts provide a _________ with ___________:

A

Nonvascularized massive allografts provide a biologic spacer with strong cortical bone.

How well did you know this?
1
Not at all
2
3
4
5
Perfectly
8
Q

Nonvascularized massive allografts: Advantages and disadvantages

A

These grafts give great strength to the construct.

Despite this they have many disadvantages:

  • lack of blood supply
  • lack of osteogenic cells
  • potential for immunologic reaction.
How well did you know this?
1
Not at all
2
3
4
5
Perfectly
9
Q
A 67-year-old man comes to the office because of an open wound of the right groin and exposed distal anastomosis of a vascular graft 3 weeks after aorto-femoral bypass grafting. Reconstruction with a sartorius muscle flap is planned to cover the graft. The blood supply of the sartorius muscle flap originates directly from which of the following arteries?
A) Deep circumflex iliac
B) Lateral circumflex femoral
C) Medial circumflex femoral
D) Profunda femoris
E) Superficial femoral
A

E) Superficial femoral

The sartorius muscle has a Type IV vascularization pattern consisting of 8 to 10 pedicles from the superficial femoral vessel, which enter the muscle medially.

How well did you know this?
1
Not at all
2
3
4
5
Perfectly
10
Q

The sartorius muscle has a Type ____ vascularization pattern

A

The sartorius muscle has a Type IV vascularization pattern, consisting of 8 to 10 pedicles

How well did you know this?
1
Not at all
2
3
4
5
Perfectly
11
Q

Vessel that supplies the sartorius muscle flap

A

The sartorius muscle has a Type IV vascularization pattern consisting of 8 to 10 pedicles from the superficial femoral vessel, which enter the muscle medially.

How well did you know this?
1
Not at all
2
3
4
5
Perfectly
12
Q
A 70-year-old man undergoes open reduction and internal fixation of an open fracture of the ankle. Debridement of nonviable tissue results in the exposure of the joint and hardware. Coverage with a reverse sural artery flap is planned. A photograph is shown. Which of the following is the most likely adverse outcome associated with the use of this flap?
A) Infection
B) Insufficient bulk
C) Insufficient flap coverage
D) Partial flap loss
E) Wound dehiscence
A

D) Partial flap loss

The utility of the flap has been proven in both healthy and compromised wounds; neither peripheral arterial disease nor diabetes has precluded its success. Of the possible complications, partial flap loss has occurred most commonly.

Cutaneous paddles have been harvested with dimensions of up to 12 × 15 cm, allowing coverage of most ankle and heel wounds. Reconstruction of larger donor defects with split-thickness skin grafts makes dehiscence unlikely.

How well did you know this?
1
Not at all
2
3
4
5
Perfectly
13
Q

Most common complication of reverse sural artery flap

A

Of the possible complications, partial flap loss has occurred most commonly.

How well did you know this?
1
Not at all
2
3
4
5
Perfectly
14
Q

Proposed modification to reduce partial flap loss of the sural artery flap

A

Maintaining a narrow (2 cm wide) pedicle, including a cutaneous ‘tail’ along the length of the pedicle and maintaining a mesentery between the sural nerve and the deep fascia.

How well did you know this?
1
Not at all
2
3
4
5
Perfectly
15
Q

Infection rates of the reverse sural artery flap

A

Infection rates have been low, ranging from 0 to 2.5%.

How well did you know this?
1
Not at all
2
3
4
5
Perfectly
16
Q

Reverse sural artery flap: Type to conform to defects, versus coverage of a deeper defect

A

The fasciocutaneous variety of the reverse sural flap is often malleable enough to conform to most defects. When a deeper defect exists, a fasciomusculocutaneous variation of the flap has been designed.

How well did you know this?
1
Not at all
2
3
4
5
Perfectly
17
Q

Reverse sural artery flap: The fasciomusculocutaneous variation carries a portion of what muscle?

A

A fasciomusculocutaneous variation of the reverse sural artery flap has been designed to carry a portion of the gastrocnemius muscle via one of the larger proximal perforators typically found deep to the cutaneous paddle in the proximal leg.

How well did you know this?
1
Not at all
2
3
4
5
Perfectly
18
Q

Reverse sural artery flap: Cutaneous paddles of dimensions up to ________

A

Cutaneous paddles have been harvested with dimensions of up to 12 × 15 cm, allowing coverage of most ankle and heel wounds.

How well did you know this?
1
Not at all
2
3
4
5
Perfectly
19
Q

Reverse sural artery flap: Cutaneous paddles allow coverage of most wounds in what location?

A

Most ankle and heel wounds

How well did you know this?
1
Not at all
2
3
4
5
Perfectly
20
Q

Reverse sural artery flap: Reconstruction of donor site when a large cutaneous paddle is taken

A

Larger donor defects are reconstructed with with split-thickness skin grafts

How well did you know this?
1
Not at all
2
3
4
5
Perfectly
21
Q
A 33-year-old woman is brought to the emergency department after sustaining a 6 × 8-cm soft-tissue defect over the tibial tuberosity after being hit by a motor vehicle while riding her bicycle. Physical examination shows exposed bone at the proximal third of the leg. Which of the following muscle flaps is most appropriate for this patient?
A) Lateral gastrocnemius
B) Lateral soleus
C) Medial gastrocnemius
D) Medial soleus
E) Sartorius
A

C) Medial gastrocnemius

How well did you know this?
1
Not at all
2
3
4
5
Perfectly
22
Q

Gastrocnemius muscle: Which compartment ?

A

The gastrocnemius is a powerful muscle in the superficial posterior compartment of the leg

How well did you know this?
1
Not at all
2
3
4
5
Perfectly
23
Q

Gastrocnemius muscle: Actions

In which activities is it involved?

A

Acts to plantar flex the foot at the ankle joint and flex the leg at the knee joint. It is involved in standing, walking, running, and jumping.

How well did you know this?
1
Not at all
2
3
4
5
Perfectly
24
Q

Gastrocnemius muscle: Anatomy

A

The lateral head originates from the lateral condyle of the femur, while the medial head originates from the medial condyle of the femur. Its other end forms a common tendon with the soleus muscle; this tendon is known as the calcaneal tendon or “Achilles tendon” and inserts onto the posterior surface of the calcaneus.

How well did you know this?
1
Not at all
2
3
4
5
Perfectly
25
Q

Gastrocnemius muscle: Anatomical difference between its two heads

A

The medial head is longer, and its muscular fibers extend more inferiorly.

How well did you know this?
1
Not at all
2
3
4
5
Perfectly
26
Q

The gastrocnemius can be harvested as a muscular or musculocutaneous flap if the _____ is intact: Which actions will be preserved?

A

The gastrocnemius can be harvested as a muscular or musculocutaneous flap if the soleus is intact and plantar flexion of the foot will be preserved.

How well did you know this?
1
Not at all
2
3
4
5
Perfectly
27
Q

Gastrocnemius muscle: When is footdrop a possible morbidity?

A

Footdrop is possible with the use of the lateral muscle belly.

How well did you know this?
1
Not at all
2
3
4
5
Perfectly
28
Q

What are the two powerful muscles in the superficial posterior compartment of the leg?

A

The gastrocnemius and soleus muscles

How well did you know this?
1
Not at all
2
3
4
5
Perfectly
29
Q

Actions of the soleus muscle

A

With the gastrocnemius, the soleus acts to plantar flex the foot at the ankle joint. Specifically, the soleus plays an important role in standing; if not for its constant pull, the body would fall forward.

How well did you know this?
1
Not at all
2
3
4
5
Perfectly
30
Q

Which muscle of the leg is important in keeping the body from falling forward?

A

The constant pull of the soleus

How well did you know this?
1
Not at all
2
3
4
5
Perfectly
31
Q

Anatomy of the soleus muscle

A

The muscle originates from the posterior surfaces of the head of the fibula and its upper quarter, as well as the middle third of the medial border of the tibia. Its other end forms a common calcaneal tendon with the gastrocnemius muscle.

How well did you know this?
1
Not at all
2
3
4
5
Perfectly
32
Q

Potential use of the soleus when freed from the Achilles tendon:

A

The soleus muscle, when freed from its insertion on the Achilles tendon and based proximally, covers defects in the middle third of the tibia.

How well did you know this?
1
Not at all
2
3
4
5
Perfectly
33
Q

Potential use of the hemi-soleus:

A

A hemisoleus may be used to cover distal third tibia.

How well did you know this?
1
Not at all
2
3
4
5
Perfectly
34
Q

What tissue types may the sartorius flap contain?

A

The sartorius flap may be raised as a muscle or myocutaneous flap

How well did you know this?
1
Not at all
2
3
4
5
Perfectly
35
Q

The sartorius flap is based on which vessels?

A

The sartorius is based on segmental branches of the superficial femoral artery and vein.

How well did you know this?
1
Not at all
2
3
4
5
Perfectly
36
Q

What type of flap is the sartorius?

A

Type IV segmental blood supply

How well did you know this?
1
Not at all
2
3
4
5
Perfectly
37
Q

Why would the sartorius flap arc of rotation be limited?

A

Because it has a Type IV segmental blood supply, the arc of rotation both superiorly and inferiorly is limited.

How well did you know this?
1
Not at all
2
3
4
5
Perfectly
38
Q

Best indication for the sartorius flap:

A

Its best indication is for coverage of the femoral vessels.

How well did you know this?
1
Not at all
2
3
4
5
Perfectly
39
Q
A 40-year-old woman is referred by her orthopedic surgeon for reconstruction because of an exquisitely tender scar over the site of the anterolateral portal for arthroscopy of the left ankle. Examination shows a 1-cm scar over the lateral malleolus. Tinel sign is present over the scar. Sensation is diminished over the middle part of the dorsum of the foot. Which of the following nerves is most likely to have been injured during the arthroscopy?
A) Lateral malleolar
B) Medial plantar
C) Saphenous
D) Superficial peroneal
E) Sural
A

D) Superficial peroneal

In the patient described, the dermatomal distribution of numbness of the middle of the dorsum of the foot suggests injury of the superficial peroneal nerve (SPN). The SPN supplies the sensation to the middle portion of the dorsum of the foot, except for the skin in the great toe web space (which is innervated by the deep peroneal nerve).

How well did you know this?
1
Not at all
2
3
4
5
Perfectly
40
Q

Superficial peroneal: Innervation to?

A

The SPN supplies the sensation to the middle portion of the dorsum of the foot, except for the skin in the great toe web space (which is innervated by the deep peroneal nerve).

How well did you know this?
1
Not at all
2
3
4
5
Perfectly
41
Q

The lateral malleolar nerve supplies innervation to?

A

The lateral malleolar nerve is a terminal branch nerve supplying the skin of the lateral malleolus.

How well did you know this?
1
Not at all
2
3
4
5
Perfectly
42
Q

The medial plantar nerve supplies innervation to?

A

The medial plantar nerve supplies the medial three fourths of the plantar surface of the foot.

How well did you know this?
1
Not at all
2
3
4
5
Perfectly
43
Q

Course of the saphenous nerve in the distal lower extremity

A

The saphenous nerve travels to the dorsum of the foot, medial malleolus, and the area of the head of the first metatarsal.

How well did you know this?
1
Not at all
2
3
4
5
Perfectly
44
Q

At the level of the ankle, the saphenous nerve is found where?

A

At the level of the ankle, the saphenous nerve is found between the medial malleolus and the anterior tibial tendon, just lateral to the saphenous vein.

How well did you know this?
1
Not at all
2
3
4
5
Perfectly
45
Q

Sural nerve location in the distal lower extremity?

A

The sural nerve is located at an average of 7 mm posterior to the lateral malleolus

How well did you know this?
1
Not at all
2
3
4
5
Perfectly
46
Q

The sural nerve supplies sensation to:

A

The sural nerve supplies sensation to the lateral aspect of the foot.

How well did you know this?
1
Not at all
2
3
4
5
Perfectly
47
Q
A 62-year-old man is brought to the emergency department by helicopter after sustaining severe injuries to the head, neck, and right femur during a motor vehicle collision. The patient's condition is stabilized, and the femur is temporarily reduced and splint immobilized. Peripheral pulses in the right leg are not palpable and capillary refill is noted; handheld Doppler shows weak pulses. Which of the following is the most appropriate next step to establish lower extremity vascular injury in this patient?
A) CT angiography
B) Doppler ultrasonography
C) Measurement of ankle brachial index
D) Serial physical examinations
A

A) CT angiography

Because of significant collateral blood flow in the upper and lower extremities, capillary refill and handheld Doppler tones can often be found even with complete disruption of major arteries.

Although traditional angiography is known as the “gold standard” for the diagnosis of vascular injuries, it is not without its difficulties. A special suite, technicians, and physicians are needed to perform traditional angiography, and the potential for morbidity has been noted. As a result, CT angiography is fast becoming the new “gold standard” for the diagnosis of vascular injuries.

How well did you know this?
1
Not at all
2
3
4
5
Perfectly
48
Q

Traditional versus CT angiography for trauma

A

Although traditional angiography is known as the “gold standard” for the diagnosis of vascular injuries, it is not without its difficulties. A special suite, technicians, and physicians are needed to perform traditional angiography, and the potential for morbidity has been noted. As a result, CT angiography is fast becoming the new “gold standard” for the diagnosis of vascular injuries.

How well did you know this?
1
Not at all
2
3
4
5
Perfectly
49
Q

A 25-year-old man is brought to the emergency department after he was hit by a motor vehicle while walking across the street. Physical examination shows a Gustilo Type IIIB open fracture of the tibia. Reconstruction with an anterolateral thigh flap is planned. An incision between which of the following structures is the most appropriate approach to access the posterior tibial vessels?
A) Lateral malleolus and the Achilles tendon
B) Lateral malleolus and the extensor hallucis longus tendon
C) Medial malleolus and the Achilles tendon
D) Tibialis anterior and the extensor hallucis longus tendons

A

C) Medial malleolus and the Achilles tendon

The sural nerve is located at the distal leg between the lateral malleolus and the Achilles tendon.

The greater saphenous vein is located between the medial malleolus and the extensor hallucis longus tendon.

The posterior tibial vessels are located between the medial malleolus and the Achilles tendon.

The dorsalis pedis artery is located between the tendons of the tibialis anterior and the extensor hallucis longus.

How well did you know this?
1
Not at all
2
3
4
5
Perfectly
50
Q

The sural nerve is located at the distal leg between which structures?

A

The sural nerve is located at the distal leg between the lateral malleolus and the Achilles tendon.

How well did you know this?
1
Not at all
2
3
4
5
Perfectly
51
Q

The greater saphenous vein is located at the distal leg between which structures?

A

The greater saphenous vein is located between the medial malleolus and the extensor hallucis longus tendon.

How well did you know this?
1
Not at all
2
3
4
5
Perfectly
52
Q

The posterior tibial vessels are located at the distal leg between which structures?

A

The posterior tibial vessels are located between the medial malleolus and the Achilles tendon.

How well did you know this?
1
Not at all
2
3
4
5
Perfectly
53
Q

The dorsalis pedis artery is located at the distal leg between which structures?

A

The dorsalis pedis artery is located between the tendons of the tibialis anterior and the extensor hallucis longus.

How well did you know this?
1
Not at all
2
3
4
5
Perfectly
54
Q

An otherwise healthy 35-year-old man is brought to the emergency department after he collided with a parked bus while riding his motorcycle at 20 mph. On admission, he is cleared by the trauma service to undergo orthopedic exploration of an open fracture of the tibia and fibula. The surgeon achieves external fixation and begins washout of a 15-cm anterior wound with exposed denuded bone. A consultation regarding initial evaluation for soft-tissue coverage is requested. Which of the following additional findings is most likely to lead to the decision to recommend eventual below-the-knee amputation rather than reconstruction for this patient?
A) Gap of the tibial nerve of 4 cm
B) Grade 2 liver laceration
C) Ipsilateral full-thickness anterior plantar skin avulsion with associated metatarsal fractures
D) New-onset absence of the pulses in the foot following external fixation
E) Parietal contusion requiring endotracheal intubation and neurosurgical consultation

A

A) Gap of the tibial nerve of 4 cm

Avulsion of the tibial nerve is one of the two absolute indications for amputation in the context of Gustilo Type IIIB and IIIC open tibial fractures. This is because outcomes of microsurgical reconstruction of the insensate foot in these cases are very poor.

The other absolute contraindication to reconstruction is a warm ischemia time of 6 hours or greater, which the patient described does not have. The patient has just lost pulses after manipulation and fixation. This is a new finding, with very recent ischemia time. This is a potentially fixable situation, and not necessarily a contraindication to reconstruction. Recent loss of pulses could contribute to a need for amputation in the future but would not require that decision now.

Polytrauma, such as a liver laceration or an intracranial injury, can evolve into relative contraindications to reconstruction, but they are not absolute contraindications by themselves.

The loss of plantar skin and metatarsal fractures by themselves are also not an absolute contraindication to reconstruction. This is a potentially reconstructible problem that should not push the plastic surgeon to recommend amputation. The additional foot trauma can potentially be reconstructed either with a skin graft or a flap.

How well did you know this?
1
Not at all
2
3
4
5
Perfectly
55
Q

Absolute indication for amputation in the context of Gustilo Type IIIB and IIIC open tibial fractures.

A
  1. Avulsion of the tibial nerve: Outcomes of microsurgical reconstruction of the insensate foot in these cases are very poor.
  2. Warm ischemia time of 6 hours or greater, which the patient described does not have.
How well did you know this?
1
Not at all
2
3
4
5
Perfectly
56
Q
A 17-year-old boy is brought to the emergency department after sustaining a traumatic injury to the left lower extremity in a motor vehicle collision. Physical examination shows a large area of crush injury, loss of soft tissue, and open fracture of the tibia with exposed bone. The lower leg is cool to touch and pale. No distal pulses are palpable. Angiography shows transsection of the popliteal artery. Which of the following is the most appropriate Gustilo classification of this patient's fracture?
A ) Type I
B ) Type II
C ) Type IIIA
D ) Type IIIB
E ) Type IIIC
A

E ) Type IIIC

Type III fractures are greater than 10 cm and involve extensive soft-tissue damage. These injuries create difficulties in coverage of bone or fixation hardware. Gustilo Type III fractures are further subdivided into A, B, and C subtypes. Type IIIA fractureshave sufficient soft tissue to provide for bony coverage. Type IIIB fractures involve periosteal stripping and extensive tissue damage, and local soft-tissue coverage is not possible. These typically result from high-energy mechanisms, such as high-velocity gunshot wounds or significant crush injuries. Gustilo Type IIIB fractures are the most common injuries for which plastic surgeons are consulted. Gustilo Type IIIC fractures include vascular injuries that require repair. The presence of a vascular injurysignificantly increases the probability of amputation.

How well did you know this?
1
Not at all
2
3
4
5
Perfectly
57
Q

Most widely accepted method for characterizing open fractures of the lower extremity

A

The Gustilo classification is the most widely accepted method for characterizing open fractures of the lower extremity.

How well did you know this?
1
Not at all
2
3
4
5
Perfectly
58
Q

Gustilo Type I

A

Type I open fractures involve soft-tissue lacerations smaller than 1 cm.

How well did you know this?
1
Not at all
2
3
4
5
Perfectly
59
Q

Gustilo Type II

A

Type II fractures include lacerations of 1 to 10 cm, with moderate soft-tissue damage

How well did you know this?
1
Not at all
2
3
4
5
Perfectly
60
Q

Gustilo Type III

A

Type III fractures are greater than 10 cm and involve extensive soft-tissue damage. These injuries create difficulties in coverage of bone or fixation hardware.

How well did you know this?
1
Not at all
2
3
4
5
Perfectly
61
Q

Gustilo Type IIIA

A

Type IIIA fractureshave sufficient soft tissue to provide for bony coverage.

How well did you know this?
1
Not at all
2
3
4
5
Perfectly
62
Q

Gustilo Type IIIB

A

Type IIIB fractures involve periosteal stripping and extensive tissue damage, and local soft-tissue coverage is not possible. These typically result from high-energy mechanisms, such as high-velocity gunshot wounds or significant crush injuries.

How well did you know this?
1
Not at all
2
3
4
5
Perfectly
63
Q

Most common Gustily type fracture for which plastic surgeons are consulted

A

Gustilo Type IIIB fractures are the most common injuries for which plastic surgeons are consulted.

How well did you know this?
1
Not at all
2
3
4
5
Perfectly
64
Q

Gustilo Type IIIC

A

Gustilo Type IIIC fractures include vascular injuries that require repair. The presence of a vascular injurysignificantly increases the probability of amputation.

How well did you know this?
1
Not at all
2
3
4
5
Perfectly
65
Q

A 15-year-old boy is brought to the emergency department 1 hour after his left foot was severed when his leg was run over by a train. Photographs are shown (long tendon stumps that appear ripped off from their muscle bellies, and ankle disarticulation). Which of the following is the most appropriate management?
A ) Construction of a foot filet free flap
B ) Replantation and ankle fusion
C ) Revision amputation to the level of the skin edge and primary closure
D ) Split-thickness skin grafting
E ) Temporary revascularization with shunts and delayed replantation

A

C ) Revision amputation to the level of the skin edge and primary closure

The patient described has a limb amputation that includes a significant avulsion component, exemplified inthe long tendon stumps that appear ripped off from their muscle bellies, and ankle disarticulation. Avulsion amputations of the lower extremity are not suitable for replantation because of the extended neurovascular damage that is present on both ends well beyond the level of the injury. The incidence of thrombosis at the anastomosis is high. Use of vein grafts on a free flap could be considered, but, in this case, they may have to be connected as high as knee level to ensure patency. The amputated part likely has massive microscopic endothelial damage extending to terminal vessels and could only be used as a donor site for skin grafts. Therefore, there is no role for immediate or delayed replantation.

While preservation of maximum tibial length is desirable in preparing the amputation stump, prosthetic fitting for ankle disarticulations is fraught with trouble. Some tibial shortening is required to have a comfortable, reliable prosthesis. In the scenario described, tibial shortening of 2 and 4 in would have equivalent results because there is still ample length of proximal tibia available to provide the same functionality and equivalent energy expenditure. Therefore, use of a free flap to protect an additional 2 in of tibia adds no advantage for this patient. When the level of amputation is through the proximal tibia, some benefit could be found in that a minimum of 6 in of proximal tibia is available for prosthetic fitting.

How well did you know this?
1
Not at all
2
3
4
5
Perfectly
66
Q

How much tibia is necessary for a prosthetic?

A

6 inches

How well did you know this?
1
Not at all
2
3
4
5
Perfectly
67
Q
A 43-year-old man has footdrop and numbness of the left foot following reconstruction of a soft-tissue defect resulting from a Gustilo Type IIIB fracture of the proximal tibia. Which of the following pedicled flaps was most likely used for reconstruction? 
A ) Gracilis 
B ) Lateral gastrocnemius 
C ) Reversed sural artery 
D ) Sartorius 
E ) Soleus
A

B ) Lateral gastrocnemius

The pedicled gastrocnemius flap has been shown to be a reliable source of vascularized soft tissue for injuries of the distal thigh, knee, and proximal leg. The lateral or medial head may be harvested. Lateral gastrocnemius harvest risks damaging the common peroneal nerve, with an incidence of 7.7%. Medial gastrocnemius harvest, for obvious anatomical reasons, does not carry this risk.

There are few data on the morbidity of donor sites, but those studies that have been done demonstrate no functional debility at a walking gait for less than 200 m. Patients do notice difficulty standing on their toes and have slowing with variable calcaneal gait when walking fast. Forty-two percent of patients could run, 22% had pain in the donor site at rest, and 20% had pain when walking more than 200 m. Seventy percent had pain and weakness in the operated leg when attempting to run. Range-of-motion deficit existed in the operated limb, with average loss of 27% flexion and 14% extension

How well did you know this?
1
Not at all
2
3
4
5
Perfectly
68
Q

Nerve damage risk of lateral versus medial gastrocnemius flap harvest

A

Lateral gastrocnemius harvest risks damaging the common peroneal nerve, with an incidence of 7.7%. Medial gastrocnemius harvest, for obvious anatomical reasons, does not carry this risk.

How well did you know this?
1
Not at all
2
3
4
5
Perfectly
69
Q

Which nerve is at risk of damage during a lateral gastrocnemius flap?
___%

A

Lateral gastrocnemius harvest risks damaging the common peroneal nerve, with an incidence of 7.7%.

How well did you know this?
1
Not at all
2
3
4
5
Perfectly
70
Q

Morbidity following a pedicled gastrocnemius flap

A
  • Difficulty standing on their toes
  • Slowing with variable calcaneal gait when walking fast.
  • 70% percent had pain and weakness in the operated leg when attempting to run
  • Range-of-motion deficit existed in the operated limb, with average loss of 27% flexion and 14% extension
How well did you know this?
1
Not at all
2
3
4
5
Perfectly
71
Q

Morbidity of soleus flap harvest versus gastrocnemius flap harvest

A
  • Functional deficit w limited ankle flexion is more severe w/ Soleus
  • Less edema with the soleus, because it has better venous muscle pump function
How well did you know this?
1
Not at all
2
3
4
5
Perfectly
72
Q
A 62-year-old man is scheduled to undergo reconstruction of a 7-cm bone defect (shown) resulting from excision of a tumor of the distal radius. He has hypertension and has smoked one pack of cigarettes daily for 30 years. Which of the following is the most appropriate technique for reconstruction of the defect?
A ) Allograft bone graft
B ) Fibula free flap
C ) Locking fixation plate
D ) Pedicled ulna bone flap
A

B ) Fibula free flap

Composite tissue reconstruction after tumor ablation is a significant challenge to the plastic surgeon. Generally, bony defects greater than 6 cm require vascularized bone for reconstruction. For large defects, the optimum choice of bone is the vascularized fibula. The fibula also has a reliable blood supply from the peroneal artery and can be harvested with a cutaneous skin paddle. The fibula free flap is the most appropriate technique. Reconstruction with free flaps can be successfully performed in a patient with a smoking history and, in fact, is often the optimal method of reconstruction in smokers. The use of allograft cadaver bone is not indicated, especially in a patient with a history of smoking. For defects of the size shown in the photograph, the locking fixation plate would not give adequate long-term support. A local bone flap is also not a viable option.

How well did you know this?
1
Not at all
2
3
4
5
Perfectly
73
Q

Reconstruction of radial defects should generally be performed when they are ____ or greater

A

Generally, bony defects greater than 6 cm require vascularized bone for reconstruction.

How well did you know this?
1
Not at all
2
3
4
5
Perfectly
74
Q

Reconstruction of choice for large radial defects

A

For large defects, the optimum choice of bone is the vascularized fibula.

How well did you know this?
1
Not at all
2
3
4
5
Perfectly
75
Q

Blood supply of a fibular free flap

A

The fibula has a reliable blood supply from the peroneal artery and can be harvested with a cutaneous skin paddle.

How well did you know this?
1
Not at all
2
3
4
5
Perfectly
76
Q

A 70-year-old man is brought to the emergency department after he was hit by a car while walking across the street. Numerous fractures of the ribs, a nonoperative laceration of the spleen, a fracture of the right humerus, and a Gustilo Type IIIC fracture of the middle third of the left lower leg are noted. Examination shows an 8 x 7-cm soft-tissue defect over the anterior aspect with exposed bone. The patient is unable to plantar flex his left foot, and sensation is absent over the plantar aspect. Surgical exploration shows transection of the posterior tibial artery and tibial nerve. Which of the following is the most appropriate management of the injured leg?
A ) Above-knee amputation
B ) Below-knee amputation
C ) Irrigation, debridement, external fixation, free tissue transfer, and delayed repair of the nerve
D ) Irrigation,debridement, external fixation, immediate repair of the artery and nerve, and free tissue transfer
E ) Irrigation, debridement, internal fixation, immediate repair of the artery and nerve, and skin grafting

A

B ) Below-knee amputation

The treatment goal in the management of open tibial fractures and lower extremity salvage is to preserve a limb that will be more functional than an amputation. If the extremity cannot be salvaged, the goal is to maintain the maximum functional length.

Given the advanced age of the patient described, his associated injuries, and an insensate injured lower extremity, the most appropriate management of the injured leg would be a below-knee amputation.

Below-knee amputations provide better prosthetic function and require less energy for ambulation than above-knee amputations.

How well did you know this?
1
Not at all
2
3
4
5
Perfectly
77
Q

An 18-year-old man is brought to the trauma center after sustaining an injury to the right lower extremity. Examination shows an open fracture of the right tibia. Which of the following mechanisms of the injury is most likely to require the most extensive surgical debridement?
A ) A collision on the rink during which one skater runs over another
B ) A fallfrom a bicycle onto a curb after a collision with a pedestrian
C ) A fall from a 6-ft ladder onto a ceramic floor
D ) A fall from a shopping cart onto a parking lot
E ) A vehicle crash into a highway barrier while speeding

A

E ) A vehicle crash into a highway barrier while speeding

All of the mechanisms could have caused an open fracture requiring surgical intervention. Only the vehicle crash represents a high-energy injury. The approach to high-energy injuries must take into account a wider zone of injury beyond just the fracture site and skin laceration.

How well did you know this?
1
Not at all
2
3
4
5
Perfectly
78
Q
A 40-year-old man undergoes open reduction and internal fixation of an open fracture of the ankle. Debridement of nonviable tissue results in the exposure of the lateral joint and hardware. Coverage of the lateral malleolus with the flap shown is planned (reverse flow sural flap). Which of the following arteries must be intact for this flap to be viable?
A ) Anterior tibial
B ) Dorsalis pedis
C ) Lateral genicular
D ) Lateral plantar
E ) Peroneal
A

E ) Peroneal

The reverse-flow sural flap has become one of the more dependable solutions in distal third leg wound and heel reconstruction. Survival of the flap depends on intact flow into the superficialsural arterial network via perforators from the peroneal system. The largest perforator arises roughly 5 cm cephalad to the lateral malleolus and typically marks the lowest pivot point for the flap

How well did you know this?
1
Not at all
2
3
4
5
Perfectly
79
Q

Blood supply of the reverse-flow sural flap

A

Superficial sural arterial network via perforators from the peroneal system.

The largest perforator arises roughly 5 cm cephalad to the lateral malleolus and typically marks the lowest pivot point for the flap

How well did you know this?
1
Not at all
2
3
4
5
Perfectly
80
Q
A 48-year-old woman undergoes coverage of a defect with an anterolateral thigh flap. Which of the following branches of the lateral femoral circumflex artery is the most likely dominant vascular supply for the flap perforator?
A ) Ascending
B ) Descending
C ) Oblique
D ) Transverse
A

B ) Descending

In recent studies by Wong, et al, and Rozen, et al, attempts have been made to clarify the vascular perforator anatomy of theanterolateral thigh flap. It has been noted that sizeable vascular perforators may arise from the ascending, transverse, or descending branch of the lateral circumflex femoral artery. The most common supply, however, comes from the descending branch, followed by the transverse and ascending branches.

How well did you know this?
1
Not at all
2
3
4
5
Perfectly
81
Q

Specific blood supply of the ALT flap

___% of cases

A

Descending branch of the lateral circumflex femoral artery, in 85%

How well did you know this?
1
Not at all
2
3
4
5
Perfectly
82
Q
A 70-year-old man has a mildly tender, clean, 6-cm wound to the lower leg that occurred spontaneously seven months ago. Physical examination of the ankle shows a brown-red discoloration and edema of the surrounding tissue. Which of the following is the most appropriate initial management of the wound?
A ) Hyperbaric oxygen therapy
B ) Silver sulfadiazine dressing
C ) Surgical debridement
D ) Unna boot compression dressing
E ) Vacuum-assisted closure therapy
A

D ) Unna boot compression dressing

The patient described has signs and symptoms of a venous stasis ulcer, a common cause of chronic lower leg wounds. Venous valve incompetence leads to chronic venous hypertension, capillary hydrostatic pressure elevation, and leakage of fluidand proteins to the extracellular space. Oxygen transport to the tissues is impaired, causing localized cellular necrosis and ulceration.

The hallmark of treatment is compression of the edematous limb to reduce the severe interstitial edema. By decreasing the tissue pressure, oxygen delivery is enhanced and wound healing mechanisms may be gradually restored. The open wound is best treated with an absorptive and occlusive dressing such as Unna wrap. Care must be taken to avoid overcompression and arterial compromise. Unna boots can be changed weekly or more frequently as needed. After reepithelialization has occurred, compression stocking or elastic bandage wraps are essential to prevent occurrence. Compression therapy is also a cost-effective treatment. Venous insufficiency affects up to 5% of the population; more than 500,000 patients in the United States suffer from ulceration. The economic costs from treatment and lost productivity are enormous.

How well did you know this?
1
Not at all
2
3
4
5
Perfectly
83
Q

How do venous stasis ulcers develop?

A

Venous valve incompetence leads to chronic venous hypertension, capillary hydrostatic pressure elevation, and leakage of fluidand proteins to the extracellular space. Oxygen transport to the tissues is impaired, causing localized cellular necrosis and ulceration.

How well did you know this?
1
Not at all
2
3
4
5
Perfectly
84
Q

Treatment of venous stasis: Hallmark?

A

The hallmark of treatment is compression of the edematous limb to reduce the severe interstitial edema. By decreasing the tissue pressure, oxygen delivery is enhanced and wound healing mechanisms may be gradually restored.

How well did you know this?
1
Not at all
2
3
4
5
Perfectly
85
Q

Management of an open venous stasis ulcer

A
  • Compression of the edematous limb to reduce severe interstitial edema.
  • Application of an absorptive and occlusive dressing such as Unna wrap
How well did you know this?
1
Not at all
2
3
4
5
Perfectly
86
Q

The sural nerve is best localized at which of the following locations?
A ) 1 cm anterior to the fibular head
B ) 1 cm posterior to the lateral malleolus
C ) 1 cm anterior to the lateral malleolus
D ) 1 cm posterior to the medial malleolus
E ) 1 cm anterior to the medial malleolus

A

B ) 1 cm posterior to the lateral malleolus

How well did you know this?
1
Not at all
2
3
4
5
Perfectly
87
Q

Desirable features of cutaneous nerve grafts

A

(1) large fascicles with little interfascicular connective tissue,
(2) separate parallel fascicles,
(3) overall large diameter,
(4) large-caliber axons,
(5) easy accessibility, with little anatomic variation,
(6) long unbranched segments, and
(7) minimal sensory deficit.

How well did you know this?
1
Not at all
2
3
4
5
Perfectly
88
Q

What type of nerve is the sural nerve

A

Purely sensory

How well did you know this?
1
Not at all
2
3
4
5
Perfectly
89
Q

What contributes to the sural nerve?

A

Contributions from:

  • medial sural cutaneous nerve (branch of the tibial nerve)
  • lateral sural cutaneous nerve (branch of the peroneal nerve)
How well did you know this?
1
Not at all
2
3
4
5
Perfectly
90
Q

What is the diameter of the sural nerve?

A

It has a mean diameter of 3.61 mm, which is ideal for group fascicular reconstruction

How well did you know this?
1
Not at all
2
3
4
5
Perfectly
91
Q

Deficit after harvest of the sural nerve

A

It leaves minimal residual sensory deficit (the lateral foot and ankle).

How well did you know this?
1
Not at all
2
3
4
5
Perfectly
92
Q

How much length can be obtained from the sural nerve?

A

Up to 30 cm of nerve graft can be obtained.

How well did you know this?
1
Not at all
2
3
4
5
Perfectly
93
Q

Location of the distal sural nerve

A

The distal portion of the nerve is consistently found between 1 and 1.5 cm posterior to the lateral malleolus.

How well did you know this?
1
Not at all
2
3
4
5
Perfectly
94
Q

A 1-year-old boy is scheduled to undergo primary surgical reconstruction of congenital talipes equinovarus. Subsequent skin shortage and wound complications are best prevented by which of the following methods?
A ) Free fasciocutaneous flap transfer
B ) Healing via second intention
C ) Preoperative tissue expansion
D ) Split-thickness skin grafting
E ) Two-stage local flap delay and transfer

A

C ) Preoperative tissue expansion

Although the treatment of congenital talipes equinovarus (CTEV) has trended toward conservative routines of manipulation and limited surgical release (eg, Ponseti technique), surgical treatment of CTEV may be necessary in cases of delayed treatment or failure of the conservative regime. Surgical correction may be attempted via a gradual technique (Ilizarov) or as an acute correction with release of the contracted posterior and medial elements.

Many acute surgical CTEV corrections can be accomplished without skin or wound difficulties; when such difficulties are anticipated, preoperative placement of tissue expanders has proven useful in allowing primary closure of the release sites with minimal morbidity.

How well did you know this?
1
Not at all
2
3
4
5
Perfectly
95
Q

Congenital talipes equinovarus: Common name?

A

Club foot

How well did you know this?
1
Not at all
2
3
4
5
Perfectly
96
Q

When is surgical treatment of congenital talipes equinovarus indicated?

A

The treatment of congenital talipes equinovarus has trended toward conservative routines of manipulation and limited surgical release.
Surgical treatment may be necessary in cases of delayed treatment or failure of the conservative regime.

How well did you know this?
1
Not at all
2
3
4
5
Perfectly
97
Q

How to ensure primary closure following surgical management of congenital talipes equinovarus?

A

Many acute surgical CTEV corrections can be accomplished without skin or wound difficulties; when such difficulties are anticipated, preoperative placement of tissue expanders has proven useful in allowing primary closure of the release sites with minimal morbidity.

How well did you know this?
1
Not at all
2
3
4
5
Perfectly
98
Q

A 60-year-oldman comes to the office because he has had tingling and numbness on the plantar aspects of the toes of both feet for the past seven months. He says that his symptoms are more noticeable after long periods of walking or standing and are relieved by elevation of the legs. Physical examination shows weakness of flexion of the toes. Tarsal tunnel syndrome is suspected. The most likely cause of these findings is palsy of which of the following nerves?
A ) Deep peroneal nerve to flexor digitorum superficialis muscle
B ) Interosseous nerve to quadratus plantae muscle
C ) Medial plantar nerve to flexor digitorum brevis muscle
D ) Superficial peroneal nerve to interosseus muscles
E ) Tibial nerve to flexor digitorum longus muscle

A

C ) Medial plantar nerve to flexor digitorum brevis muscle

The tibial nerve bifurcates distally into the medial and lateral plantar nerves, and it is these two nerves that are impinged in the tarsal tunnel.

How well did you know this?
1
Not at all
2
3
4
5
Perfectly
99
Q

What nerve(s) can be compressed in the tarsal tunnel?

A

The tibial nerve bifurcates distally into the medial and lateral plantar nerves, and it is these two nerves that are impinged in the tarsal tunnel.

How well did you know this?
1
Not at all
2
3
4
5
Perfectly
100
Q

Actions of the flexor digitorum brevis?

A

The flexor digitorum brevis flexes both the proximal and the middle phalanges of the toes

How well did you know this?
1
Not at all
2
3
4
5
Perfectly
101
Q

Innervation of the flexor digitorum brevis

A

Medial plantar nerve

How well did you know this?
1
Not at all
2
3
4
5
Perfectly
102
Q

Intrinsic toe flexors

A
  • Flexor digitorum brevis
  • Flexor hallucis brevis
  • Flexor digiti minimi brevis
  • Interossei
  • Lumbricals
How well did you know this?
1
Not at all
2
3
4
5
Perfectly
103
Q

Innervation of the Flexor hallucis brevis

A

Medial plantar nerve

How well did you know this?
1
Not at all
2
3
4
5
Perfectly
104
Q

Innervation of the Flexor digiti minimi brevis

A

Lateral plantar nerve

How well did you know this?
1
Not at all
2
3
4
5
Perfectly
105
Q

Innervation of the Interossei of the foot

A

Lateral plantar nerve

How well did you know this?
1
Not at all
2
3
4
5
Perfectly
106
Q

Innervation of the lumbricals of the foot

A

Medial and lateral plantar nerves

How well did you know this?
1
Not at all
2
3
4
5
Perfectly
107
Q

Most severe presentation of tarsal tunnel syndrome

A

In the most severe situation, a patient with tarsal tunnel syndrome may have “claw toes.”

How well did you know this?
1
Not at all
2
3
4
5
Perfectly
108
Q

Actions of the quadratus plantae

A

Quadratus plantae is a flexor of the lateral four toes through the flexor digitorum longus

How well did you know this?
1
Not at all
2
3
4
5
Perfectly
109
Q

Innervation of the quadratus plantae

A

Lateral plantar nerve.

How well did you know this?
1
Not at all
2
3
4
5
Perfectly
110
Q

Innervation of the flexor digitorum longus

A

The flexor digitorum longus is a toe flexor and is innervated by the tibial nerve

How well did you know this?
1
Not at all
2
3
4
5
Perfectly
111
Q

A 34-year-old man who works as a police officer is brought to the emergency department after sustaining a traumatic avulsion of the right heel. Examination shows a 3 * 3-cm area of exposed calcaneus on the weight-bearing aspect of the heel. Which of the following arteries and nerves supply the flap that will provide the most appropriate sensate coverage?
Artery || Nerve
A )Dorsalis pedis || superficial peroneal
B )Medial plantar || medial plantar
C )Medial sural || saphenous
D )Posterior tibial || posterior tibial
E )Radial || medial antebrachial cutaneous

A

B )Medial plantar || medial plantar

Sensate flaps are beneficial in resurfacing wounds located on weight-bearing areas as in pressure sores or the heel, as in the scenario described.

The most reliable sensate flap for coverage of the plantar calcaneus comes from the medial plantar flap. This flap receives its sensation from the medial plantar nerve (L4-5) and receives its blood supply from the medal plantar artery. This flap comes from the instep of the foot between the head of the first metacarpal and the midpoint of the heel. The size of the flap can be up to 12 * 6 cm. Due to its proximity to the heel and the minimal donor morbidity with loss of sensation to the instep of the foot, this flap is the most appropriate for coverage of this patient’s defect

How well did you know this?
1
Not at all
2
3
4
5
Perfectly
112
Q

How are sensate flaps performed (reinnervation)

A

Sensate flaps include an intact sensory nerve supplying the overlying skin being transferred or, as in the case of a free tissue transfer, the cutaneous nerve is preserved and reanastomosed to a recipient cutaneous nerve.

113
Q

What is the most reliable sensate flap to cover the plantar calcaneus?

A

The most reliable sensate flap for coverage of the plantar calcaneus comes from the medial plantar flap, due to its proximity to the heel and the minimal donor morbidity with loss of sensation to the instep of the foot

114
Q

Medial plantar flap: Sensory innervation

A

This flap receives its sensation from the medial plantar nerve (L4-5)

115
Q

Medial plantar flap: Blood supply

A

Medial plantar artery

116
Q

Medial plantar flap: Comes from where?

A

This flap comes from the instep of the foot between the head of the first metacarpal and the midpoint of the heel.

117
Q

Medial plantar flap: Greatest size

A

The size of the flap can be up to 12 * 6 cm.

118
Q

Medial plantar flap: Donor site morbidity

A

Loss of sensation to the instep of the foot

119
Q

Blood supply of the dorsalis pedis flap

A

The dorsalis pedis flap receives its blood supply from the dorsalis pedis artery.

120
Q

Sensory innervation of the dorsalis pedis flap

A

This flap can be transferred as a sensate flap using the superficial peroneal nerve.

121
Q

The dorsalis pedis flap is most frequently used for:

A

The dorsalis pedis flap is used most frequently for coverage of the anterior ankle and dorsal distal foot

122
Q

Blood supply of the medial gastrocnemius flap

A

The medial sural artery supplies the medial gastrocnemius flap.

123
Q

The medial gastrocnemius flap is best used for:

A

This flap is best used for soft-tissue coverage of the upper third of the leg and the distal knee.

124
Q

The medial gastrocnemius flap: sensory innervation

A

Preservation of the saphenous nerve with the medial gastrocnemius musculocutaneous flap will provide a neurosensory flap.

125
Q

Medial soleus muscle flap: Nerve

A

The proximal branches of the posterior tibial artery supply the medial soleus muscle flap.

126
Q

Medial soleus muscle flap: Sensory innervation

A

The posterior tibial nerve supplies only motor fibers to the soleus muscle and can therefore not be used as a sensate flap.

127
Q

Medial soleus muscle flap: Best use

A

This flap is best used for soft-tissue coverage of the middle third of the leg.

128
Q

Blood supply of the radial forearm flap

A

The radial forearm flap receives its blood supply from the radial artery and its septocutaneous perforating vessels.

129
Q

Radial forearm flap: Use for reconstruction of the plantar surface of the foot

A

A neurosensory flap can be harvested based on the medial or lateral antebrachial cutaneous nerves to reconstruct the plantar surface of the foot. This would be a second choice to the medial plantar artery flap as it requires free tissue transfer

130
Q

A 52-year-old man is brought to the emergency department after sustaining a gunshot wound to the right forearm. History includes well-controlled diabetes mellitus type 2, coronary artery disease, and renal insufficiency. Examination of the forearm shows viable muscle coverage and gross instability. A radiograph is shown (large defect of the radius). Following debridement and stabilization of the wound, reconstruction of the defect is planned via a vascularized free fibular transfer. In addition to clinical examination, which of the following is the most appropriate preoperative evaluation of this patient’s lower extremity?
A ) Ankle brachial indices
B ) Color-flow Doppler imaging
C ) CT angiography
D ) Traditional angiography
E ) Transcutaneous partial pressure of oxygen

A

B ) Color-flow Doppler imaging

While anatomic variants in the vasculature of the leg are rare, failure to recognize these prior to sacrifice of the peroneal artery during harvest of a fibular free flap can result in disastrous ischemic complications. Variations occur more commonly in patients who have vascular insufficiency due to underlying atherosclerotic changes. In these patients, clinical examination alone is insufficient to adequately define blood flow patterns to the leg and foot.

Color-flow Doppler imaging has demonstrated excellent ability to define the presence of flow-limiting lesions in the leg. Monophasic signals in any of the three major runoff vessels strongly suggest that the limb will be at risk for ischemia following sacrifice of the peroneal artery. Comparison of the findings on color-flow Doppler with angiography demonstrates that angiography is an unnecessary addition to the initial study.

Angiography and CT angiography offer excellent road maps of the vasculature of the lower extremity, but each does involve the use of intravenous contrast. The dose of contrast used in CT angiography is significantly lower than that in traditional angiography, but neither would be justified in this patient with pre-existing renal insufficiency.

MR angiography provides a map of the vessels comparable to both angiography and CT angiography without the use of nephrotoxic contrast agents. The additional information gained by seeing the vessels preoperatively (by MRA, CT angiography, or traditional angiography) may be justified when very long segments of the bone need to be harvested or when the perfusion to the overlying skin paddle may be better defined.

Transcutaneous partial pressure of oxygen measures perfusion and does not offer any anatomic detail.

131
Q

When should further investigation for anatomic variants in vascularity be performed, prior to sacrifice of the peroneal artery for a free fibula?

A

Variations occur more commonly in patients who have vascular insufficiency due to underlying atherosclerotic changes. In these patients, clinical examination alone is insufficient to adequately define blood flow patterns to the leg and foot.

While anatomic variants in the vasculature of the leg are rare, failure to recognize these prior to sacrifice of the peroneal artery during harvest of a fibular free flap can result in disastrous ischemic complications.

132
Q

What result from color-flow doppler imaging would be concerning, prior to sacrifice of the peroneal artery, for a free fibula flap?

A

Color-flow Doppler imaging has demonstrated excellent ability to define the presence of flow-limiting lesions in the leg. Monophasic signals in any of the three major runoff vessels strongly suggest that the limb will be at risk for ischemia following sacrifice of the peroneal artery.

133
Q

Assessing the peroneal artery vasculature prior to fibula free flap: Color-flow Doppler versus Angiography

A

Comparison of the findings on color-flow Doppler with angiography demonstrates that angiography is an unnecessary addition to the initial study.

134
Q

Free fibula osseocutaneous flap: Dominant arterial inflow

A

Peroneal artery

135
Q

Proximally based soleus muscle flap: Dominant arterial inflow

A

Popliteal artery

136
Q

Distally based soleus muscle flap: Dominant arterial inflow

A

Posterior tibial artery

137
Q
A 17-year-old boy sustains a traumatic injury to the right lower extremity. Examination shows weakness of plantar flexion and loss of sensation over the plantar surface of the foot. Which of the following nerves is most likely injured?
A ) Deep peroneal
B ) Femoral
C ) Obturator
D ) Superficial peroneal
E ) Tibial
A

E ) Tibial

The most likely nerve to be injured is the tibial nerve.

The tibial nerve is a branch of the sciatic nerve. It travels through the popliteal fossa and gives off branches to the gastrocnemius, soleus, plantaris, and popliteus muscles. The tibial nerve travels in proximity to the posterior tibial artery. In the leg, it gives off branches to the flexor digitorum longus, tibialis posterior, and flexor hallucis longus. Distally in the foot, it branches to give rise to the medial and lateral plantar nerves, which provide sensation to the plantar surface of the foot. Injury to the tibial nerve results in deficits of plantar flexion, as well as anesthesia to the plantar surface of the foot. Tibial nerve disruption has been considered an indication for amputation as opposed to limb salvage in traumatic injuries of the lower extremity. However, one study suggests that limb salvage can be performed even in the face of absent plantar sensation at the time of presentation. This may indicate thatabsence of plantar sensation is not a reliable correlate for tibial nerve disruption.

138
Q

The tibial nerve is a branch of:

A

The tibial nerve is a branch of the sciatic nerve

139
Q

Anatomy / supply of the tibial nerve in the popliteal fossa

A

It travels through the popliteal fossa and gives off branches to the:

  • gastrocnemius
  • soleus
  • plantaris
  • popliteus muscles.
140
Q

Anatomy / supply of the tibial nerve in the leg

A

The tibial nerve travels in proximity to the posterior tibial artery. In the leg, it gives off branches to the:

  • flexor digitorum longus
  • tibialis posterior
  • flexor hallucis longus.
141
Q

Anatomy of the tibial nerve most distally

A

Distally in the foot, it branches to give rise to the medial and lateral plantar nerves, which provide sensation to the plantar surface of the foot.

142
Q

Tibial nerve injury: Implications on the foot

A

Injury to the tibial nerve results in deficits of plantar flexion, as well as anesthesia to the plantar surface of the foot.

143
Q

The deep peroneal nerve arises from the ____________ at the ____________

A

The deep peroneal nerve arises from the common peroneal nerve at the fibular neck.

144
Q

The deep peroneal nerve travels in which compartment of the leg?

A

It travels in the anterior compartment of the leg

145
Q

The deep peroneal nerve supplies which muscles?

A

It gives branches to the:

  • tibialis anterior
  • extensor hallucis longus
  • extensor digitorum longus
  • extensor digitorum brevis
  • peroneus tertius
146
Q

Sensory supply of the deep peroneal nerve?

A

The sensory distribution is in the area of the first web space.

147
Q

Injury to the deep peroneal nerve causes:

A

Injury to the deep peroneal nerve causes weakness in dorsiflexion of the foot.

148
Q

The femoral nerve innervates which muscles?

A

The femoral nerve innervates muscles of the anterior thigh, including the quadriceps group, iliacus, and sartorius.

149
Q

Injury to the femoral nerve results in:

A

Injury to the femoral nerve results in weakness of leg extension.

150
Q

The superficial peroneal nerve arises from the _________ at the __________

A

The superficial peroneal nerve arises from the common peroneal nerve at the fibular neck.

151
Q

The superficial peroneal nerve supplies which muscles?

A

It supplies the lateral compartment of the leg, giving motor branches to peroneus longus and brevis

152
Q

The superficial peroneal nerve supplies sensation where?

A

Sensory contribution to the lateral aspect of the leg.

153
Q

Injury to the superficial peroneal nerve results in:

A

Injury to the superficial peroneal nerve results in anesthesia of the lateral aspect of the leg and weakness in eversion and plantar flexion of the foot.

154
Q

The obturator nerve provides innervation to which muscles?

A

The obturator nerve provides innervation to the medial thigh muscles (adductor group), including:

  • Adductor brevis, longus, and magnus,
  • Gracilis
  • Obturator externus.
155
Q

Which branch of the obturator nerve supplies sensation?

To where?

A

The cutaneous branch provides sensation of the medial thigh.

156
Q

Injury to the obturator nerve results in:

A

Injury to the obturator nerve results in weakness in thigh adduction and sensory deficits in the medial thigh.

157
Q

A 19-year-old man is brought to the emergency department after sustaining a heavily contaminated, open fracture of the tibia during an all-terrain vehicle collision. The injury is classified as a Gustilo Type IIIB fracture of the tibia in the distal third of the right leg. A comminuted 4-cm segment of the tibia is debrided. A soft-tissue defect measuring 7 * 4 cm overlies the fracture site. Which of the following is the most appropriate initial method to stabilize the fracture?
A ) Contralateral free fibula flap with intramedullary rod stabilization
B ) Iliac crest bone graft with plate stabilization
C ) Ipsilateral pedicled fibula flap with intramedullary rod stabilization
D ) Placement of an antibiotic-impregnated spacer
E ) Placement of an external fixation device

A

E ) Placement of an external fixation device

The most appropriate initial step for fracture stabilization is placement of an external fixation device. The patient described will require multiple debridements for the heavy contamination, followed by a free muscle flap to provide soft-tissue coverage of the exposed tibia fracture, and a future avascular bone graft for the 4-cm segmental bone loss.

Intramedullary rod stabilization for the initial management of open fractures of the tibia has gained increasing popularity in the orthopaedic literature, but the bone fixation is converted from external fixation to an intramedullary rod within the first 10 days of injury immediately prior to muscle flap coverage

A 4-cm segmental bone loss does not require vascularized bone by either a pedicled or free bone flap.

158
Q
A 38-year-old man comes to the office for follow-up examination two months after undergoing open reduction and internal fixation of a fracture of the right proximal fibula. A preoperative radiograph is shown (Fibular head fracture). On current physical examination, he can evert the right foot, but he is unable to dorsiflex it. This patient has most likely sustained an injury to which of the following nerves?
A ) Common peroneal 
B ) Deep peroneal 
C ) Sciatic 
D ) Superficial peroneal 
E ) Sural
A

B ) Deep peroneal

The patient described has sustained an injury to his deep peroneal nerve. An electromyelogram should be ordered to confirm the diagnosis. The common peroneal nerve divides into two branches. The deep branch of the peroneal nerve innervates the extensor hallucis longus and the anterior tibial muscles. The superficial branch of the peroneal nerve provides motor innervation to the peroneus longus and brevis muscles. If the common peroneal nerve had been injured, eversion would not have been likely. A sural nerve injury would have a sensory component, not a motor component. The sciatic nerve injury would have more global consequences, though this pattern is theoretically possible in a Sunderland Grade VI injury. Thus, the most likely injured nerve is the deep peroneal nerve.

159
Q

Branches of the common peroneal nerve, and what they supply

A

The common peroneal nerve divides into two branches:

  1. ) The deep branch of the peroneal nerve innervates:
    - extensor hallucis longus
    - the anterior tibial muscles.
  2. ) The superficial branch of the peroneal nerve provides motor innervation to:
    - peroneus longus and brevis
160
Q

A 42-year-old man with type 1 diabetes mellitus comes to the office because he has had a sore on the heel of the left foot for the past 12 months. Physical examination of the foot shows a 4 * 4-cm ulcer. Pulses are difficult to palpate on the dorsalis pedis and posterior tibial arteries. No other abnormalities are noted. Noninvasive vascular studies are performed. Reconstruction with a free flap is planned. Which of the following is most likely to indicate the need for major arterial inflow reconstruction prior to reconstruction of the defect?
A ) Ankle-brachial index of less than 0.7
B ) Biphasic arterial Doppler waveform
C ) Delay in capillary refill of the toes of more than 3 seconds
D ) Doppler signal loss from the dorsalis pedis artery
E ) Great toe pressure of less than 30 mmHg

A

E ) Great toe pressure of less than 30 mmHg

Thirty percent of diabetic patients have significant arterial calcifications in these arteries, which impedes blood pressure cuff compression, rendering the ankle-brachial pressure index inaccurate for ischemia. The ankle-arm index of 0.7 may thus be falsely elevated in this patient. Ordinarily, one would attempt to raise this index to at least 0.5 or 0.6 before performing soft-tissue reconstruction. Biphasic or triphasic arterial waveforms are usually consistent with sufficient vascularity for wound healing. Monophasic waveforms are associated more with extremity ischemia. However, even monophasic waveforms, as long as there is good amplitude and a narrow waveform complex, may be acceptable prior to attempting soft-tissue reconstruction.

Digital vessels are frequently spared from calcification and absolute toepressures are a better indicator of peripheral ischemia than ankle pressures. A toe pressure of less than 30 mmHg indicates ischemia and the need to obtain an angiogram with a view to prior vascular inflow improvement (such as angioplasty, stenting, or surgical reconstruction) before proceeding to soft-tissue reconstruction. Capillary refill time is a subjective clinical evaluation that is not as accurate or objective as pressure testing. Incomplete plantar arch communication between the dorsalis pedis andposterior tibial arterial systems is not an accurate indication for ischemia.

161
Q

ABI and accuracy, in diabetic patients

A

30% of diabetic patients have significant arterial calcifications in their arteries, which impedes blood pressure cuff compression, rendering the ankle-brachial pressure index inaccurate for ischemia.

162
Q

Lower extremity wound healing: Waveforms consistent with sufficient vascularity for wound healing, vs insufficient waveforms

A

Biphasic or triphasic arterial waveforms are usually consistent with sufficient vascularity for wound healing. Monophasic waveforms are associated more with extremity ischemia. However, even monophasic waveforms, as long as there is good amplitude and a narrow waveform complex, may be acceptable prior to attempting soft-tissue reconstruction.

163
Q

Goal minimum ABI for lower extremity reconstruction

A

Ordinarily, one would attempt to raise this index to at least 0.5 or 0.6 before performing soft-tissue reconstruction.

164
Q

Best indicator measurement of peripheral ischemia in lower extremity wounds

A

Digital vessels are frequently spared from calcification and absolute toepressures are a better indicator of peripheral ischemia than ankle pressures.

165
Q

A toe pressure of less than _______ indicates ischemia and the need to obtain:

A

A toe pressure of less than 30 mmHg indicates ischemia and the need to obtain an angiogram with a view to prior vascular inflow improvement (such as angioplasty, stenting, or surgical reconstruction) before proceeding to soft-tissue reconstruction.

166
Q
A 55-year-old man who is employed as a police officer is brought to the emergency department after sustaining injuries to the left lower extremity during a motor vehicle collision. Physical examination shows fracture of the tibia with 2 cm of bone loss, a large amount of tissue loss, and soft-tissue contamination extending over the calf, ankle, and forefoot. Sensation is absent in the plantar surface. Which of the following factors is most likely to influence the surgeon to proceed with immediate amputation of the leg?
A ) Age of patient
B ) Amount of bone loss
C ) Contamination of soft tissue
D ) Injury to posterior tibial nerve
A

D ) Injury to posterior tibial nerve

In recent publications from the Lower Extremity Assessment Project (LEAP), a multicenter prospective study to determine outcomes following lower extremity trauma, the most significant factor in a surgeon’s decision to immediately amputate was the absence of plantar sensation and the severity of soft-tissue injury. While age, soft-tissue contamination, and bone loss may be significant predictors of long-term limb salvage and function, they were not as significant in impacting the surgeon’s initial decision to salvage or amputate the leg.

167
Q

A 50-year-old man is scheduled to undergo repair of a Gustilo grade IIIB open fracture of the tibia and fibula, resulting in an 8-cm segmental defect of the tibia. Which of the following is an advantage of a vascularized fibula graft over a nonvascularized autograft for this patient?
(A)Greater bone resorption
(B)Greater osteocyte viability
(C)Incorporation by creeping substitution
(D)Remodeling
(E)Replacement of necrotic bone

A

(B)Greater osteocyte viability

Compared with a conventional nonvascularized autograft, a vascularized fibula graft has increased osteocyte viability. In one study, osteocyte counts in the cortex of vascularized grafts were 89% of those observed in normal control subjects. There were substantially fewer surviving osteocytes in conventional grafts.

Nonvascularized bone autograft incorporation occurs by creeping substitution, which involves gradual vascular ingrowth, resorption, and replacement of necrotic bone. The revascularization process that occurs causes significant mechanical weakening due to bone resorption at six to 12 months.

The transfer of vascularized bone grafts increases cell survival and as a result the process of bone graft incorporation by creeping substitution is obviated. Significant osteopenia is not seen and bone mass is maintained after transfer. The result isimproved strength, healing, and stress response. Cell survival results in less remodeling during revascularization.

168
Q

Nonvascularized bone autograft incorporation occurs by:

A

Nonvascularized bone autograft incorporation occurs by creeping substitution, which involves gradual vascular ingrowth, resorption, and replacement of necrotic bone. The revascularization process that occurs causes significant mechanical weakening due to bone resorption at six to 12 months.

169
Q

The transfer of vascularized bone grafts increases _______, and as a result what is obviated?

A

The transfer of vascularized bone grafts increases cell survival and as a result the process of bone graft incorporation by creeping substitution is obviated. Significant osteopenia is not seen and bone mass is maintained after transfer. The result isimproved strength, healing, and stress response. Cell survival results in less remodeling during revascularization

170
Q

A 35-year-old man comes to the office for evaluation of purulent drainage from the surgical wound one year after open reduction and internal fixation of tibia and fibula fractures sustained in a motorcycle collision. Radiographs show osteomyelitis and nonunion of tibia and fibula fractures. He is taken to the operating room for debridement of scarred and fibrotic soft tissue, removal of hardware, and application of external fixation. A cutaneous scapula free flap is used to close an 8-cm defect. Intravenous antibiotics are initiated and continued for six weeks. Three months later, the purulent drainage recurs. Which of the following is the most likely cause of treatment failure?
(A)Flap closure was under tension
(B)Inadequate antibiotic therapy
(C)Inadequate bone debridement
(D)Use of a cutaneous scapula free flap rather than a muscle free flap
(E)Use of a one-stage rather than a two-stage procedure

A

(C)Inadequate bone debridement

Lower extremity osteomyelitis following trauma is a challenging problem with high complication rates. The key to success is adequate debridement of both bone and soft tissue followed by coverage with vascularized tissue. The patient described haddebridement of soft tissue but not bone. The bone is a persistent source of infection and the scarred fibrotic soft tissue is poorly vascularized, thus crippling the immune response. Without adequate bone debridement, early recurrence is likely. Muscle flaps fill the resultant defect, enhance blood flow, and improve immunologic defense against microorganisms.

171
Q

An 18-year-old man comes to the office for follow-up examination because he has a four-month history of tripping with numbness and tingling of the right leg and foot. He underwent arthroscopic anterior cruciate ligament (ACL) grafting five months ago after he dislocated the right knee and ruptured the ACL during a football game. Physical examination shows weakness of dorsiflexion of the foot. Decreased sensibility to light touch over the lateral aspect of the right leg is noted. Which of the following is the most likely cause of his symptoms?
(A)Compartment syndrome
(B)Compression of the common peroneal nerve
(C)Laceration of the deep peroneal nerve
(D)Laceration of the superficial peroneal nerve
(E)Neurapraxia of the tibial nerve

A

(B)Compression of the common peroneal nerve

Compression of the common peroneal nerve is the third most common nerve compression syndrome (carpal tunnel syndrome is first and cubital tunnel syndrome is second) and the most common nerve compression syndrome in the lower extremity. Knee dislocations are a common cause of common peroneal nerve compression. The patient described has a chronic injury that has not caused complete paralysis orloss of sensibility. The initial injuries would likely cause scar tissue around the knee but are not sharp in nature. Compression of the common peroneal nerve would result in weakness in the anterior compartment muscles caused by slowing of conduction in the deep peroneal nerve and paresthesias to the superolateral foot caused by slowing of conduction in the superficial peroneal nerve.

172
Q

3 Most common nerve compression syndromes

A
  1. Carpal tunnel
  2. Cubital tunnel
  3. Compression of the common peroneal nerve
173
Q

Compression of the common peroneal nerve would result in:

A
  • weakness in the anterior compartment muscles caused by slowing of conduction in the deep peroneal nerve
  • paresthesias to the superolateral foot caused by slowing of conduction in the superficial peroneal nerve.
174
Q

A 24-year-old man is brought to the emergency department by ambulance because of injuries to the right leg he sustained in a motorcycle collision. Physical examination shows fracture of the tibia and fibula, thrombosis of the anterior and posterior tibial arteries, and transection of the posterior tibial and peroneal nerves just below the knee. Which of the following is the most appropriate management?
(A) External fixation
(B) Skeletal traction and vacuum-assisted closure of the wound
(C) Revascularization and coverage with a gastrocnemius flap
(D) Revascularization and coverage with a latissimus dorsi flap
(E) Below-the-knee amputation

A

(E) Below-the-knee amputation

This patient has sustained multiple traumas including injury to the skin, bone, arteries, and nerves. Using the injury severity scale, the prognosis of this injury is very poor. In an adult, there is little chance of regaining useful protective sensation or motor function after nerve repair in light of the open fractures and vascular injury. The best immediate management is amputation. Additionally, the return to weight-bearing and work is significantly shorter with amputation.

175
Q
A 14-year-old girl comes to the office for follow-up examination seven days after undergoing intramedullary nail fixation of a fracture of the right tibia. Along with bony stabilization, the degloved skin from the posterior middle and distal third of the leg was tacked back in place. On physical examination, necrosis of the replaced skin is noted. After debridement, the tibia is exposed in the middle to distal third of the leg. Which of the following is the most appropriate management?
(A) Split-thickness skin graft
(B) Sural artery flap
(C) Soleus flap
(D) Free latissimus flap
(E) Amputation
A

(D) Free latissimus flap

This Gustilo IIIB wound is best covered with a free muscle flap. The latissimus muscle flap is an effective and frequently used flap for large wounds. This flap also has the advantage of large vessels and a long pedicle length.

Dressing changes followed by a skin graft will not provide durable coverage over the tibia, is not appropriate in a healthy teenager who is not at surgical risk, and the time delay to coverage may predispose the patient to the development of osteomyelitis. A neuroadipofascial flap based on the vessels accompanying the sural nerve would be a poor choice, as this would be harvested from the zone of injury. The soleus flap is traditionally used for middle third defects of the leg and also is within the zone of injury. This is a salvageable extremity in a young teenager, which makes amputation inappropriate.

176
Q
Which of the following anatomic structures is an important landmark in raising a reverse sural artery flap?
(A) Achilles tendon
(B) Deep peroneal nerve
(C) Lesser saphenous vein
(D) Plantaris tendon
(E) Posterior tibial artery
A

(C) Lesser saphenous vein

When raising a reverse sural artery flap, the important landmarks are the lesser saphenous vein and sural nerve, which should bisect the cutaneous paddle. The blood supply to this flap depends on the medial superficial sural artery and the lesser saphenous vein with its two small accompanying arteries. The pivot point of the pedicle is typically 5 cm above the lateral malleolus, where the perforators of the flap enter a more superficial plane

177
Q

When raising a reverse sural artery flap, the important landmarks are:

A

When raising a reverse sural artery flap, the important landmarks are the lesser saphenous vein and sural nerve, which should bisect the cutaneous paddle.

178
Q

Blood supply of a reverse sural artery flap

A

The blood supply to this flap depends on the medial superficial sural artery and the lesser saphenous vein with its two small accompanying arteries.

179
Q

Pivot point of a reverse sural artery flap

A

The pivot point of the pedicle is typically 5 cm above the lateral malleolus, where the perforators of the flap enter a more superficial plane

180
Q

The deep peroneal nerve is located in the ________ compartment.

A

The deep peroneal nerve is located in the lateral compartment.

181
Q
A 53-year-old man with a comminuted fracture of the midtibia has a 4 x 3-cm defect of the midanterior surface of the leg at the level of the fracture. He currently smokes two packs of cigarettes daily. Physical examination shows no palpable dorsalis pedis pulse. Which of the following surgical interventions is the most appropriate method of reconstruction in this patient?
(A) Anterior tibialis muscle flap 
(B) Below-knee amputation
(C) Gastrocnemius muscle flap
(D) Gracilis free tissue transfer 
(E) Soleus muscle flap
A

(E) Soleus muscle flap

The soleus muscle flap is most appropriate for reconstruction in this patient. The soleus is a bipenniform muscle; its medial head originates from the posterior tibia, and the lateral head originates from the proximal fibula. It is located deep to the gastrocnemius in the superficial posterior compartment. Blood to the medial head is predominantly supplied by the popliteal and posterior tibial arteries and the lateral head is predominantly supplied by the peroneal artery. Depending on the size of the defect, a hemisoleus muscle flap can be used to preserve flexor function.

Free tissue transfer is often used for reconstruction of high-velocity injuries to avoid the use of muscle in the zone of injury. Free tissue transfer, however, is not the best option for this 53-year-old man because his history of cigarette smoking and absent pedal pulse suggest the possibility of peripheral vascular disease.

182
Q

Blood supply to the soleus muscle flap

A

Blood to the medial head is predominantly supplied by the popliteal and posterior tibial arteries
Blood to the lateral head is predominantly supplied by the peroneal artery.

183
Q

Options for lower-extremity reconstruction:

  • Knee wounds and proximal tibial defects:
  • Middle third defects:
  • Distal third defects:
A

For lower-extremity reconstruction:

  • Knee wounds and proximal tibial defects: the gastrocnemius muscle flap
  • Middle third defects: the soleus
  • Distal third defects: free tissue transfer
184
Q
A 53-year-old man with diabetes mellitus has a nonhealing wound over the right calcaneus. A bone scan shows increased uptake at the site of the wound. Which of the following is the most definitive diagnostic test?
(A) Bone biopsy and culture
(B) CT scan
(C) Indium scan
(D) MRI
(E) Wound biopsy and culture
A

(A) Bone biopsy and culture

Although osteomyelitis may be suspected clinically, definitive diagnosis is accomplished with bone biopsy and culture.

185
Q
A 27-year-old man develops osteomyelitis after sustaining an open fracture of the distal tibia and fibula in a motorcycle accident. Following bony debridement, there is a 9-cm segmental loss of the distal tibia. An external fixator is applied, and a muscle flap is to be used to cover the skin defect. Which of the following is the most appropriate management of the bony defect?
(A) Free fibula flap
(B) Iliac crest bone grafting
(C) Papineau bone grafting
(D) Tibiofibular synostosis
(E) Ilizarov bone lengthening
A

(A) Free fibula flap

The most appropriate management of this patient’s bone defect is a free fibula surrounding flap. Management typically depends on the volume of the bony defect along with soft-tissue stability and vascularity. If the tissue is unstable and poorly vascularized, a regional flap or free flap is required. For most bone defects smaller than 6 cm, traditional tricortical iliac crest bone grafts can be placed beneath the muscle flap. In patients with larger defects, a vascularized bone flap, such as the free fibula flap, is required. Vascularized bone also minimizes the risk for nonunion. However, weight-bearing ambulation should not occur for many months after flap coverage to allow for healing of bone without infection.

186
Q

Tibial defects: Management of defects 6 cm

A

For most bone defects smaller than 6 cm, traditional tricortical iliac crest bone grafts can be placed beneath the muscle flap. In patients with larger defects, a vascularized bone flap, such as the free fibula flap, is required.

187
Q

The saphenous artery originates from which ofthe following arteries?
(A) Descending lateral femoral circumflex
(B) Genicular
(C) Profunda femoris
(D) Superficial femoral

A

(B) Genicular

The saphenous artery supplies vascularity to the saphenous flap, a fasciocutaneous flap that provides only limited coverage of small defects in the region of the knee. The saphenous artery is a branch of the descending genicular artery. The genicular artery arises from the superficial femoral artery.

188
Q

The saphenous artery is a branch of:

A

The saphenous artery is a branch of the descending genicular artery.

189
Q

The genicular artery arises from the:

A

The genicular artery arises from the superficial femoral artery.

190
Q

The descending branch of the lateral femoral circumflex artery arises from the ___________ and continues distally in the leg as the ___________

A

The descending branch of the lateral femoral circumflex artery arises from the profunda femoris and continues distally in the leg as the superior lateral genicular artery.

191
Q

The profunda femoris artery gives off its major branches where?

A

The profunda femoris artery gives off its major branches more proximally in the leg.

192
Q

The superficial femoral artery becomes the ___________ at the ___________

A

The superficial femoral artery becomes the popliteal artery at the popliteal fossa.

193
Q

Which of the following vascular structures supply the secondary vascular pedicle of the gracilis muscle flap?
(A) Lateral femoral circumflex artery and vein
(B) Medial femoral circumflex artery and vein
(C) Obturator artery and vein
(D) Profunda femoris artery and vein
(E) Superficial femoral artery and vein

A

(E) Superficial femoral artery and vein

The gracilis flap is a type II muscle flap (one dominant pedicle and one secondary pedicle) that is used for pedicled and free tissue reconstructive procedures. This flap is comprised of one primary vascular pedicle, which is supplied by the profunda femoris artery via the ascending branch of the medial femoral circumflex artery and vein, and one secondary vascular pedicle, which is supplied by the superficial femoral artery and vein.

The lateral femoral circumflex artery and vein supply the rectus femoris and vastus lateralis muscle flaps.The obturator artery is a branch of the internal iliac artery and divides further to give off vascular branches, primarily in the thigh.

194
Q

What type of flap is the gracilis?

A

The gracilis flap is a type II muscle flap (one dominant pedicle and one secondary pedicle)

195
Q

Pedicles of the gracilis flap?

A

This flap is comprised of one primary vascular pedicle, which is supplied by the profunda femoris artery via the ascending branch of the medial femoral circumflex artery and vein, and one secondary vascular pedicle, which is supplied by the superficial femoral artery and vein.

196
Q

The lateral femoral circumflex artery and vein supply what flaps?

A

The lateral femoral circumflex artery and vein supply the rectus femoris and vastus lateralis muscle flaps.

197
Q

The obturator artery is a branch of what?

A

The obturator artery is a branch of the internal iliac artery and divides further to give off vascular branches, primarily in the thigh.

198
Q
A 28-year-old man has a markedly diminished dorsalis pedis pulse on the right after being stabbed with an ice pick in the right lower leg. In this patient, the most likely finding on physical examination is numbness at which ofthe following sites?
(A) Dorsal foot
(B) First web space
(C) Lateral foot
(D) Medial foot
(E) Plantar foot
A

(B) First web space

This 28-year-old man has a markedly diminished dorsalis pedis pulse after sustaining a laceration of the anteriortibial artery when he was stabbed in the leg. Therefore, he is most likely to have numbness of the first web space of the foot on physical examination. The first web space is innervated by the deep peroneal nerve; this nerve courses within the anterior compartment of the leg with the anterior tibial artery, which becomes the dorsalis pedis artery in the foot.The dorsal aspect of the foot (with the exception of the first web space) is innervated by the superficial peroneal nerve. The lateral foot is innervated by the sural nerve, and the medial foot is innervated by the saphenous nerve. None of these nerves travels with an artery.The plantar surface of the foot is innervated by the tibial nerve, which courses within the deep posterior compartment of the leg with the posterior tibial artery.

199
Q

The first web space of the foot is supplied by?

A

The deep peroneal nerve

200
Q

Course of the deep peroneal nerve

A

This nerve courses within the anterior compartment of the leg with the anterior tibial artery.

201
Q

The anterior tibial nerve becomes what in the foot?

A

The dorsalis pedis artery

202
Q

The dorsal aspect of the foot is innervated by?

A

The dorsal aspect of the foot (with the exception of the first web space) is innervated by the superficial peroneal nerve.

203
Q

The lateral aspect of the foot is innervated by?

A

The lateral foot is innervated by the sural nerve

204
Q

The medial aspect of the foot is innervated by?

A

The medial foot is innervated by the saphenous nerve.

205
Q

The plantar aspect of the foot is innervated by?

A

The plantar surface of the foot is innervated by the tibial nerve.

206
Q

Which of the following is the most common primary arterial blood supply to the great toe flap used in toe-to-thumb transfer?
(A) First dorsal metatarsal artery from the communicating branch between the dorsalis pedis and plantar arteries
(B) First dorsal metatarsal artery from the dorsalis pedis artery
(C) First dorsal metatarsal artery from the plantar arch
(D) Second dorsal metatarsal artery from the dorsalis pedis artery
(E) Second dorsal metatarsal artery from the plantar arch

A

(B) First dorsal metatarsal artery from the dorsalis pedis artery

The first dorsal metatarsal artery (FDMA), arising from the dorsalis pedis artery, is the most common primary arterial blood supply to the great toe flap, which is used in toe-to-thumb transfer. Although the vascular anatomy that supplies the great toe flap is somewhat variable, the FDMA typically arises from the dorsalis pedis artery and travels in a relatively dorsal plane. It may travel dorsal or plantar to the interosseous muscle or through the muscle itself.The FDMA is less likely to arise from the communicating branch between the dorsalis pedis and plantar arteries or from the plantar arch.The second dorsal metatarsal artery does not contribute to the great toe flap

207
Q

Most common blood supply to the great toe flap

A

The first dorsal metatarsal artery (FDMA), arising from the dorsalis pedis artery, is the most common primary arterial blood supply to the great toe flap

208
Q
A 77-year-old man has a 4 x 3-cm defect of skin and soft tissue over the distal third of the tibia after he had a stroke and fell. He has a history of myocardial infarction and chronic obstructive pulmonary disease. Physical examination of the lower leg shows exposed bone and desiccated periosteum; there is no fracture. Which of the following is most appropriate for reconstruction of the defect?
(A) Full-thickness skin graft
(B) Cross-leg flap
(C) Fasciocutaneous flap
(D) Gastrocnemius flap
(E) Free tissue transfer
A

(C) Fasciocutaneous flap

The most appropriate management is coverage of the defect with a fasciocutaneous flap. This flap is ideal for reconstruction of lower extremity wounds in patients with severe illness or multiple trauma, or in patients with small wounds that cannot be covered with a skin graft alone. The fasciocutaneous flap can be based either proximally or distally on various septocutaneous perforators, including those of the medial leg (which lie approximately 3 cm posterior to the tibia), the posterolateral septum, and the anterolateral leg.

209
Q

A 50-year-old man has noninfected wound dehiscence with exposure of the hardware two weeks after undergoing open reduction and internal fixation of an ankle fracture.. Following debridement of the wound, which of the following is the most appropriate management?
(A) Skin grafting
(B) Coverage with a free flap
(C) Application of an external fixator and skin grafting
(D) Removal of the hardware and skin grafting
(E) Removal of the hardware and coverage with a free flap

A

(B) Coverage with a free flap

This patient has early wound dehiscence after undergoing open reduction and internal fixation of a lateral malleolus fracture. Appropriate management involves early coverage of the exposed bone and hardware to promote bony union while preventing bacterial colonization of the hardware, which may lead to the development of osteomyelitis. Because this patient underwent open reduction and internal fixation only two weeks ago, fixation is still required to maintain rigid fracture stabilization. Instead, coverage with a free flap is recommended to provide stability and vascularity and to enhance function once bony union is obtained.

210
Q

An ambulatory 75-year-old man has a 6-cm grade IV pressure ulcer on the sacrum. The most appropriate management is reconstruction with which of the following?
(A) Bilateral gluteal myocutaneous advancement flaps
(B) Unilateral superior/inferior gluteal myocutaneous rotation flap
(C) Unilateral superior gluteal artery perforator flap
(D) Thin split-thickness skin graft

A

(C) Unilateral superior gluteal artery perforator flap

The unilateral superior gluteal artery perforator flap is most appropriate for reconstruction of the area of the sacrum affected by the pressure ulcer. Perforator flaps can be based on either the superior or inferior gluteal artery. They provide adequate myocutaneous coverage and at the same time preserve the gluteal muscles on the involved side. As a result, morbidity at the donor site will be minimized, which is especially important in this patient because he is ambulatory.

Although a bilateral superior/inferior gluteal myocutaneous advancement flap supplies sufficient soft-tissue coverage for pressure ulcers, harvest of the inferior and superior gluteal muscles will result in significant morbidity at the donor site. Harvest of a gluteal myocutaneous rotationflap sacrifices muscle function and also is not the procedure of choice in an ambulatory patient. Thin split-thickness skin grafts do not provide stable coverage of pressure ulcers.

211
Q

Reconstruction of a stage IV pressure ulcer on the sacrum, in an ambulatory patient

A

The unilateral superior gluteal artery perforator flap is most appropriate for reconstruction of the area of the sacrum affected by the pressure ulcer:

  • Adequate myocutaneous coverage
  • Preserve the gluteal muscles on the involved side.

Morbidity at the donor site will be minimized, which is especially important in an ambulatory patient

212
Q

The unilateral superior gluteal artery perforator flap: Blood supply

A

Based on either the superior or inferior gluteal artery.

213
Q

A 78-year-old man has an infection of the groin wound after undergoing femoral popliteal artery bypass grafting. A sartorius muscle flap is to be used for coverage of the exposed prosthetic graft. Which of the following arteries provide the blood supply for this flap?
(A) Lateral femoral circumflex artery and superficial femoral artery
(B) Lateral femoral circumflex artery and deep femoral artery
(C) Medial circumflex femoral artery and superficial circumflex iliac artery
(D) Medial circumflex femoral artery and deep femoral artery
(E) Superficial circumflex iliac artery and superficial femoral artery

A

(E) Superficial circumflex iliac artery and superficial femoral artery

The superficial circumflex iliac and superficial femoral arteries provide vascularity to the sartorius muscle flap. The superficial circumflex iliac artery is a branch of the external iliac artery; it courses laterally to provide the primary blood supply to the groin flap and the proximal part of the sartorius muscle. The external iliac artery then continues as the common femoral artery and branches into the profunda femoris and superficial femoral arteries at a point distal to the inguinal ligament. The superficial femoral artery travels distally in the thigh and gives off segmental branches to the sartorius muscle.

214
Q

Sartorius muscle flap: Blood supply

A
  • Superficial circumflex iliac

- Superficial femoral arteries

215
Q

The superficial circumflex iliac artery is a branch of the:

A

The superficial circumflex iliac artery is a branch of the external iliac artery

216
Q

The external iliac artery continues in the leg as the __________ and branches into:

A

The external iliac artery continues as the common femoral artery and branches into the profunda femoris and superficial femoral arteries at a point distal to the inguinal ligament.

217
Q

A 27-year-old man has a 12-cm defect of the tibia after sustaining open comminuted fractures of the tibia and fibula in a motorcycle accident. On physical examination, sensation is intact. The wounds are irrigated and debrided, and an external fixator is applied. Which of the following is most appropriate for reconstruction of the defect?
(A) Bone allograft
(B) Contralateral free vascularized fibula graft
(C) Free vascularized iliac crest graft
(D) Ipsilateral pedicle fibula graft
(E) Nonvascularized iliac crest graft

A

(B) Contralateral free vascularized fibula graft

In a patient who has a 12-cm bone defect, the most appropriate management is reconstruction using a well-vascularized, uninjured segment of bone graft, such as a contralateral free vascularized fibula graft. The contralateral fibula will provide more than 20 cm of bony length. Neither allograft nor nonvascularized autograft would provide the required bone length for revascularization. For this reason, nonvascularized bone graft is also not appropriate.

Vascularized iliac crest graft can be used for reconstruction in the upper and lower extremities but would not provide a large enough segment of bone graft.

Because of this patient’s injuries, the ipsilateral fibula should not be used. In addition, harvest of an ipsilateral bone graft would remove the ipsilateral supplemental support.

218
Q

The free fibula graft can provide how much length?

A

More than 20 cm

219
Q
Six months after undergoing plate fixation and primary wound closure for management of open fractures of the distal tibia and fibula sustained in a motorcycle accident, a 43-year-old man has purulent drainage from the wound site. Radiographs show bony nonunion at the fracture sites. Which of the following is the most appropriate initial management?
(A) Continuous irrigation
(B) Debridement of bone
(C) Coverage with a muscle flap
(D) Bone grafting
(E) Insertion of an intramedullary rod
A

(B) Debridement of bone

Osteomyelitis is a frequent complication of open fractures associated with soft-tissue injury, fibrosis, and localized ischemia. Measures to prevent the development of osteomyelitis, including removal of dead and devitalized bone, closure of dead space, and coverage with well-vascularized soft tissue, are recommended. In patients with established osteomyelitis, the most appropriate initial management is debridement of devascularized bone and necrotic or scarred tissues and removal of any nonautologous material, such as fixation devices.

220
Q

A 29-year-old woman is brought to the emergency department one hour after sustaining a traumatic avulsion amputation at the level of the proximal third of the lower leg. On examination, the tibia is exposed, and there is not adequate soft tissue for coverage of the bone. The amputated part has been appropriately preserved; examination shows extensive injury in the middle third of the leg but no evidence of injury in the distal third or the foot. In order to provide the best functionaloutcome in this patient, which of the following is the most appropriate management?
(A) Above-knee amputation
(B) Immediate coverage of the exposed tibia with a reverse rectus femoris flap
(C) Immediate coverage of the exposed tibia with a filet-of-foot free flap
(D) Immediate coverage of the exposed tibia with a latissimus dorsi muscle flap and split-thickness skin graft
(E) Microvascular replantation of the amputated leg with 12 cm of bone shortening to allow for complete soft-tissue coverage

A

(C) Immediate coverage of the exposed tibia with a filet-of-foot free flap

The most appropriate management is immediate coverage of the exposed tibia using a filet-of-foot free flap. Free tissue from the amputated foot can be transferred to provide appropriate soft-tissue coverage without additional donorsite morbidity. The dorsalis pedis artery and/or posterior tibial artery supply vascularity to the flap, and the tibial nerve provides sensory innervation.

221
Q

Filet of foot free flap: Vascular supply

A

The dorsalis pedis artery and/or posterior tibial artery supply vascularity to the flap

222
Q

Filet of foot free flap: Sensory innervation

A

The tibial nerve

223
Q
A 34-year-old motorcycle driver sustains a Gustilo type IIIB open fracture of the right tibia. At the time of debridement, stabilization, and coverage of the wound, there is a superficial infection of the tibia. Low-pressure pulse lavage is to be performed.Which of the following solutions will most likely preserve osteoblast-osteoclast function while clearing bone-adherent bacteria?
(A) 1% Bacitracin
(B) 1% Chlorhexidine
(C) 1% Ethanol
(D) 1% Povidone-iodine
(E) 1% Surgical soap
A

(E) 1% Surgical soap

According to the results of an animal in vitro study, low-pressure pulse lavage with 1% surgical soap resulted in a more significant clearing of bone-adherent bacteria than other solutions. Low-pressure lavage was defined as 14 psi of pressure with 550 pulsations per minute; the solutions were evaluated in 1% and 10% concentrations. Osteoclast and osteoblast function was also shown to be preserved more significantly with use of the 1% soap or detergent solution. In contrast, the other solutions studied were shown to have a deleterious effect on osteoclastand osteoblast function. Low-pressure lavage with chlorhexidine or povidone-iodine was associated with the greatest decrease in osteoblast and osteoclast function.

High-pressure lavage should not be used because it results in a significant depth or zone of bony injury.

224
Q

Pulse lavage: Which solutions have the greatest vs least effect on osteoclast and osteoblast function?

A

Most preserved osteoblast/osteoclast function: 1% Surgical soap

Least preserved osteoblast/osteoclast function: Chlorhexidine and povidone-iodine

225
Q

How would surgical soap act on bacteria in osteomyelitis?

A

The antibacterial mechanism of soap is thought to act through its micelle function. The hydrophilic ends of the soap micelle surround the bacteria, interfering with its potential for adherence to bone.

226
Q

Low pressure pulse lavage is defined as:

A

14 psi of pressure with 550 pulsations/minute

227
Q

A 24-year-old woman has a 3-cm soft-tissue defect at the medial malleolus 10 months after sustaining an open fracture of the distal third of the tibia in a motorcycle accident. There were no vascular or nerve injuries at the time of the accident. Examination shows exposed bone with dense scarring of the skin and adherence to underlying tissue. Radiographs are consistent with osteomyelitis and fracture nonunion. Which of the following is the most appropriate management?
(A) Total contact casting
(B) Split-thickness skin grafting
(C) Coverage of the wound with a soleus pedicle flap
(D) Coverage of the wound with a free latissimus dorsi flap
(E) Below-knee amputation

A

(D) Coverage of the wound with a free latissimus dorsi flap

The most appropriate management of this patient with a soft-tissue defect and osteomyelitis is stabilization of the segment, debridement of the affected area, and coverage with a free latissimus dorsi pedicle flap. Application of an external fixator, radical sequestrectomy, placement of antibiotic beads, and autotransplantation of a free latissimus dorsi flap are performed initially; bone grafting of the tibial defect is performed secondarily.

A soleus pedicle flap would not reliably cover this complex distal leg wound. In addition, the posterior compartment leg muscles and their vascular supply are frequently injured by high-energy injuries. Therefore, the soleus muscle is not likely to be reliable for transfer. A below-knee amputation would not be appropriate in a healthy patient with a sensate distal extremity in which the circulation is fully intact.

228
Q

The basic principles for use of free flaps for lower extremity soft tissue defect and osteomyelitis include:

A

The basic principles for use of free flaps for lower extremity soft tissue defect and osteomyelitis include:

  • Skeletal stabilization
  • Radical debridement of infected or devitalized tissues
  • Coverage with vascularized tissues. The free flap provides an abundance of vascularized tissue necessary to close large wounds left by aggressive debridement, and the muscle free flap provides pliability to fill the dead spaces.
229
Q
A 42-year-old man sustains open fractures of the tibia and fibula (Gustilo type IIIB) when he is thrown from his motorcycle. The wounds are irrigated, debrided, and covered with a free flap.Which of the following will NOT increase this patient's risk for the development of osteomyelitis?
(A) Dead space at the surgical site
(B) Inadequate soft-tissue coverage
(C) Internal fixation
(D) Presence of nonviable muscle
(E) Retention of devitalized bone
A

(C) Internal fixation

Patients who sustain open fractures of the long bones are susceptible to bacterial invasion of the wound site and the subsequent development of infection, especially osteomyelitis. Factors associated with persistent infection include the presence of dead space at the surgical site, inadequate soft-tissue coverage, the presence of nonviable soft tissue, including muscle, and retention of devitalized bone. Aggressive irrigation and debridement should be performed in anypatient with an open fracture. All nonviable tissue must be removed, and the fracture site should be covered with adequate soft tissue from a local site or via a free tissue transfer.

Internal fixation has not been associated with development of osteomyelitis in patients who have open fractures of the tibia and/or fibula.

230
Q

Internal fixation vs development of osteomyelitis in patients who have open fractures of the tibia and/or fibula.

A

Internal fixation has not been associated with development of osteomyelitis in patients who have open fractures of the tibia and/or fibula.

231
Q
The percentage of persons who have absence of the plantaris muscle in one lower extremity is 
(A) less than 5%
(B) 10% to 15%
(C) 25% to 30%
(D) 45% to 50%
(E) 75% to 80%
A

(B) 10% to 15%

According to the results of several studies, the plantaris muscle has been shown to be absent in at least one lower extremity in approximately 10% to 15% of persons. This tendon can be used for grafting procedures. It provides a longer donor graft than the palmaris longus in the forearm; however, it is hidden on physical examination, making it more difficult to locate prior to surgery. Because the muscle may be absent in one or both lower extremities, ultrasonography can be helpful in determining its presence and location. Approximately 33% of patients have a plantaris muscle in only one lower extremity.

232
Q

The plantaris muscle has been shown to be absent in at least one lower extremity in approximately ___% of persons.

A

According to the results of several studies, the plantaris muscle has been shown to be absent in at least one lower extremity in approximately 10% to 15% of persons.

233
Q

Because the plantaris muscle may be absent in one or both lower extremities, what can be helpful in determining its presence and location?

A

Because the muscle may be absent in one or both lower extremities, ultrasonography can be helpful in determining its presence and location.

234
Q

Approximately ___% of patients have a plantaris muscle in only one lower extremity.

A

Approximately 33% of patients have a plantaris muscle in only one lower extremity.

235
Q

The plantaris muscle is more likely absent in which lower extremity?

A

The left

236
Q

The above photograph is of a 45-year-old man with insulin-dependent diabetes mellitus who develops a gangrenous toe. Culture of the wound shows mixed aerobic and anaerobic organisms, including Bacteroides, Enterococcus, and Staphylococcus. Noninvasive vascular studies show an ankle-brachial index of 0.76. The patient wishes to undergo a single-stage surgical procedure. Which of the following is the most appropriate type of amputation for this patient?
(A) Amputation at the level of the metatarsophalangeal joint
(B) Transmetatarsal amputation
(C) Lisfranc amputation
(D) Syme’s amputation
(E) Below-knee amputation

A

(B) Transmetatarsal amputation

Amputation is indicated in this patient who has obvious necrosis of the toe. Indeed, ischemic changes and wound problems are often seen in patients with diabetes mellitus. Because these patients are typically predisposed to further, more proximal amputations in the future, a conservative approach to amputation should be used in this instance.

237
Q

An ankle-brachial index of __________ indicates a markedly increased risk for wound healing problems following surgery.

A

An ankle-brachial index of less than 0.7 indicates a markedly increased risk for wound healing problems following surgery.

238
Q

A 37-year-old man who has paraplegia to the level of T9 is referred for evaluation because he has had a chronic penetrating wound on the weight-bearing surface of the exposed left calcaneus for the past 10 years. He has also had several associated episodes of cellulitis that have resolved with conservative therapy consisting of wound care, elevation of the limb, and administration of antibiotics. In order to exclude a diagnosis of osteomyelitis of the calcaneus, which of the following is the most appropriate diagnostic study?
(A) Measurement of erythrocyte sedimentation rate
(B) Plain radiographs of the heel
(C) High-resolution CT scan
(D) MRI
(E) Triple-phase bone scan

A

(D) MRI

An MRI should be performed in this patient to exclude a diagnosis of osteomyelitis. Although all of the imaging studies listed above will show abnormalities of bone, only a gadolinium-enhanced MRI will delineate the signal characteristics of bone marrow and specific enhancement pattern characteristic of osteomyelitis.

Measurement of erythrocyte sedimentation rate is highly sensitive but nonspecific for osteomyelitis; this test is used instead as a follow-up for determining the efficacy of treatment in patients previously diagnosed with osteomyelitis.

239
Q

Best imaging test for diagnosis of osteomyelitis

A

MRI

240
Q
A 65-year-old attorney has severe ischemia of the right leg. On examination, the leg is gangrenous and ulcerated; he has pain with motion and at rest. Noninvasive vascular studies show an ankle-brachial index of 0.16. He refuses to undergo amputation. Which of the following is the most appropriate technique for limb salvage?
(A) Distal arterial bypass
(B) Distal venous arterialization bypass
(C) Endovascular stent placement
(D) Free muscle transfer
(E) Lumbar sympathectomy
A

(B) Distal venous arterialization bypass

Because both the pedal and crural arteries are occluded in this patient, neither direct arterial bypass nor placement of an endovascular stent will address the problem.

The symptoms and findings of severe pain at rest, ulceration, and gangrene seen in this patient are indicative of limb ischemia, a critically urgent condition that results from occlusion of the pedal and crural arteries. Although amputation had been performed in the past for patients with this condition, distal venous arterialization bypass is now a recommended alternative method for limb salvage. According to one small study of 18 patients, limb salvage was successfully accomplished in 83% of patients at surgery and 75% of patients at follow-up examination one year later. In these patients, the distal bypass was performed to the venous vessels of the foot using a conduit of either vein, synthetic graft, or a combination of both. The valves of the venous system were destroyed, and arterial inflow was then provided by the most distal patent artery.

241
Q

Symptoms of limb ischemia

A

The symptoms and findings of severe pain at rest, ulceration, and gangrene are indicative of limb ischemia

242
Q
A 35-year-old man with insulin-dependent diabetes mellitus develops a 14 * 8-cm ulcer of the left posterior calf overlying the Achilles tendon. He underwent kidney transplantation two years ago and has been taking immunosuppressive agents since that time. Which of the following is the most appropriate management?
(A) Growth factor therapy
(B) Skin grafting
(C) Free tissue transfer
(D) Syme's amputation
(E) Below-knee amputation
A

(C) Free tissue transfer

This 35-year-old man with diabetes mellitus who previously underwent kidney transplantation has a large ulcer overlying the left Achilles tendon. Because of his immunosuppressive state and concomitant diabetes mellitus, he is most likely to experience cardiovascular problems, neuropathy, and poor wound healing following any type of reconstructive surgery. However, despite these potential adverse sequelae, free tissue transfer is most appropriate for management of the ulcer. Appropriate work-up should be obtained before attempting any reconstructive procedure.

243
Q

Amputations and cardiac considerations in diabetic patients

A

Because amputations often increase cardiovascular demand in patients with diabetes mellitus, they should only be used as a last resort.

244
Q

An otherwise healthy 65-year-old man is evaluated because of a 2-month history of a nonhealing wound to the back of the left heel. He has a history of smoking 50 packs of cigarettes yearly but quit 1 year ago. Physical examination shows a clean wound with exposed bone and palpable distal pulses in the lower extremities. Coverage with a distally based fasciocutaneous sural flap is planned. Because of the patient’s history of smoking, a “delay” procedure is performed first. Division of which of the following is required for this procedure?
A) Distal greater saphenous vein
B) Distal lesser saphenous vein
C) Perforator 5 cm proximal to the lateral malleolus
D) Proximal greater saphenous vein
E) Proximal lesser saphenous vein

A

E) Proximal lesser saphenous vein

The surgical step required as part of the “delay” procedure in a distally based sural flap is division of the proximal lesser saphenous vein. The distally based sural flap is a neurofasciocutaneous flap used to reconstruct ankle, heel, and foot defects. The classically described and possibly most important arterial supply to the distally based sural flap is provided by septocutaneous perforators arising from the peroneal artery. The most distal of these is located 4 to 7 cm proximal to the lateral malleolus. However, there are at least three other sources described: fasciocutaneous perforators from the posterior tibial artery, venocutaneous perforators from the lesser saphenous vein, and neurocutaneous perforators from the sural nerve. The skin and fascia of the flap are drained primarily by the lesser saphenous vein. The lesser saphenous vein contains numerous valves that prevent retrograde blood flow. There are, however, one or more smaller collateral veins that run parallel to the lesser saphenous vein. These veins have anastomotic connections to the lesser saphenous vein, which can allow blood to bypass the valves of the lesser saphenous vein and flow in a retrograde fashion.

In attempts to redirect blood flow and decrease the risk of flap necrosis and other complications, several authors have described sural flap delay procedures. Two distinct delay procedures have been described. In one, the flap is first elevated without completely incising the proximal edge of the skin island. A powder-free glove is then placed between the elevated fascia and the gastrocnemius muscle, and the skin is closed. Two weeks later, the flap is completely elevated and transferred into the defect site. This procedure has the goal of redirecting blood flow in a longitudinal direction before complete elevation of the flap. In the other technique, the flap is raised in its entirety and then sutured back into its donor site. The flap is then transferred into its recipient site as a second procedure. This technique allows the flap to become viable on its distal vascular pedicle before causing the additional trauma of transferring the flap, which can potentially compromise that pedicle.

Division of the greater saphenous vein is not indicated because it is not in the vicinity of the flap. Similarly, division of the perforator 5 cm proximal to the lateral malleolus is not appropriate because this is the major pedicle supplying the flap.

245
Q

A 56-year-old man who is an active smoker sustains a degloving injury of the left foot from a motorcycle collision. The heel was avulsed from the calcaneus by a deep posterior laceration but has normal capillary refill. No tissue is missing, but the wound is heavily contaminated and the calcaneus has an abrasion that is imbedded with grit. After irrigation and debridement in the operating room, which of the following is the most appropriate next step in management of this wound?
A) Coverage with a free gracilis muscle flap
B) Healing by secondary intention
C) Layered closure over a drain
D) Negative pressure wound therapy and skin grafting
E) Serial debridement and delayed closure

A

E) Serial debridement and delayed closure

The most appropriate management of this wound is serial debridement and delayed closure. With such a high level of contamination of both the soft and hard tissues, layered closure after the initial debridement will very likely lead to infection, especially in a patient with a history of smoking. It would be a mistake to perform a free tissue transfer in a highly contaminated wound. Furthermore, there is no missing or ischemic tissue. Negative pressure wound therapy followed by skin grafting would not be appropriate for a deep wound with bone exposure when local tissues are available for closure; this would be more appropriate for a superficial wound with missing skin. Healing by secondary intention is an option; however, serial debridement and delayed closure will take less time, is less painful, and avoids scar formation in the heel.

246
Q

A thin 40-year-old woman has an 8 × 5-cm skin defect in the distal third of the anterior leg extending to the dorsum of the foot, with tibia denuded of periosteum and exposed tendon, after undergoing stabilization of the fracture with internal hardware 3 days ago. The distal posterior tibial artery was ligated before surgery at the distal third of the leg. There are no signs of infection or osteomyelitis. Which of the following is the most appropriate method of reconstruction?
A) Application of bilaminate neodermis (Integra) and negative pressure wound therapy
B) Coverage with a dorsalis pedis flap
C) Coverage with a free anterior lateral thigh (ALT) flap
D) Coverage with a free tranverse rectus abdominus myocutaneous (TRAM) flap
E) Coverage with a reverse sural flap

A

C) Coverage with a free anterior lateral thigh (ALT) flap

A free anterior lateral thigh flap is large enough to close the defect, can be thinned for aesthetics and shoe wear, and may allow for primary closure of the donor site. Although free tranverse rectus abdominus myocutaneous (TRAM) flap coverage is a possibility, the potential complications of taking muscle and unpredictable control of the final contour make them less ideal options. The reverse sural flap is not an option because of the ligation of the posterior tibial artery. In addition to having severe donor site morbidity, the dorsalis pedis would remove the remaining blood supply to the foot. The vascular nature of the defect’s wound bed makes bilaminate neodermis (Integra) and negative pressure wound therapy a less optimal choice.

247
Q

A 15-year-old girl sustained an isolated open tibial fracture in a motor vehicle collision. At the proximal third of the tibia, 15 cm of anterior soft-tissue loss is noted. Despite fracture reduction, the foot is warm but pulseless without dopplerable signals. The patient is otherwise stable. Which of the following is the most appropriate next step in management?
A) Below-knee amputation
B) CT angiography
C) Four-compartment fasciotomy
D) Internal fixation and soft-tissue coverage
E) Surgical exploration of the popliteal artery

A

B) CT angiography

Lower extremity fractures with combined soft-tissue and neurovascular trauma have high rates of complications, and a percentage of these injuries lead to amputation. Risk factors for amputation include Gustilo IIIC injuries, sciatic or tibial nerve injuries, prolonged ischemia (more than 4 to 6 hours), significant soft-tissue injury, significant wound contamination, multiple injured extremities, advanced age, lower versus upper extremity trauma, and futile attempt at revascularization. While tibial nerve injury is a risk factor and relative indication for amputation, it is never an absolute indication for amputation.

Hard signs for vascular injury include: active hemorrhage, expanding hematoma, bruit or thrill, absent distal pulses, and distal ischemic signs and symptoms (five P’s). In the face of these hard signs, imaging such as CT angiography should be used to evaluate for vascular injury. With that said, most hard signs can be explained by soft-tissue or bone bleeding, traction of intact arteries due to unreduced fractures, or compartment syndrome.

Early soft-tissue coverage is associated with a lower complication rate. The goal is to close wounds within 7 to 10 days to decrease the risk for infection, osteomyelitis, nonunion, and further tissue loss.

It is best to get wound control prior to bone grafting, avoiding the risk of losing valuable limited bone; therefore, bone grafting is generally postponed until 8 to 10 weeks after soft-tissue wound coverage.

248
Q

A 20-year-old man has purulent breakdown 5 months after sustaining a Gustilo type IIIB open fracture treated with intramedullary rod placement and skin grafting over a medial gastrocnemius flap. A postoperative x-ray study and current photograph are shown. The intramedullary rod is removed and an external fixator is placed. There is 1.5 cm of bone without periosteum surrounding the fracture exposed in the wound. Which of the following is the most appropriate next step in wound reconstruction?

A) Full-thickness skin grafting with a bolster dressing
B) Reconstruction with an anterior tibial artery perforator flap
C) Reconstruction with a lateral gastrocnemius muscle flap and skin grafting
D) Reconstruction with a pedicled descending medial genicular artery flap
E) Split-thickness skin grafting with negative pressure wound therapy

A

B) Reconstruction with an anterior tibial artery perforator flap

Perforator flap reconstruction, whether free or pedicled, has become increasingly popular over the past decade. Perforator flap use allows for the creation of an axial pattern flap without the sacrifice of a major artery and can often be done for areas once considered to require free flaps for coverage. Prior transfer of a medial gastrocnemius flap might disrupt perforators from the posterior tibial artery to the medial leg skin, but would not have disturbed anterior tibial artery perforators through the skin of the anterolateral leg.

Skin grafting, whether split- or full-thickness, would not be successful on fractured bone without periosteum, regardless of the type of dressing used.

The lateral gastrocnemius muscle is smaller and cannot reach as far as the medial gastrocnemius. It would not be able to reach the mid-shaft tibia defect shown in this patient.

The descending medial genicular artery is the pedicle of the medial femoral condyle flap. It is normally used as a bone graft donor, although an overlying skin paddle can be harvested with it. When used in a pedicled fashion, it can be transposed proximally onto the thigh, but not distally onto the leg.

249
Q

A 35-year-old man is referred to the office after undergoing prolonged failed attempts at local wound care of an exposed Achilles tendon. Physical examination shows that the tendon is beginning to desiccate. Coverage with a flap is performed, as shown. The blood supply to this flap is derived from which of the following arteries? Reverse sural fasciocutaneous flap

A) Anterior tibial
B) Geniculate
C) Peroneal
D) Popliteal
E) Superficial femoral
A

C) Peroneal

The flap used to cover the Achilles tendon in the patient described is the reverse sural fasciocutaneous flap. It is based on perforators from the peroneal artery through a network of small vessels. The general axis of the flap follows the sural nerve from behind the lateral malleolus to the mid portion of the gastrocnemius muscle bellies. Sural nerve injury results in loss of lateral foot sensation. This is often of no functional consequence because its harvest does not result in loss of a major neurovascular structure.

The other vessels noted do not supply inflow to the flap.

250
Q
A 66-year-old man comes for evaluation because of a chronic left lower extremity wound. He reports that the wound has been present for the past 15 years. Physical examination shows an 8 × 10-cm wound on the lateral aspect of the left calf. Which of the following is the most appropriate next step in diagnosis?
A) Angiography
B) Biopsy
C) Bone scan
D) CT scan
E) MRI
A

B) Biopsy

The most important next step in establishing a diagnosis in this patient is an excisional biopsy. The clinical appearance and duration of the chronic wound is highly suggestive of a Marjolin ulcer in the setting of a chronic venous stasis ulcer. Marjolin ulcers are defined as malignant generation in the presence of a burn wound or other chronic inflammatory conditions. The most common etiology is a burn wound; however, malignancies have been found in chronic wounds secondary to pressure ulceration, trauma, venous stasis, and others. The most common pathologic diagnosis is well-differentiated squamous cell carcinoma (SCC), but basal cell carcinoma and various other sarcomas have been reported in the literature. Marjolin ulcers are thought to be aggressive forms of SCC with metastatic potential related to tumor grade. The incidence of metastasis is 10% for Grade 1 lesions, 59% for Grade 2, and 86% for Grade 3.

Diagnosis is based on clinical appearance, history, and most importantly biopsy. Patients should also receive a thorough regional lymph node exam, CT scan or MRI, and routine laboratory analysis. Sentinal lymph node biopsy may be indicated. Treatment is generally wide excision of the chronic wound and rapid coverage with skin grafts or tissue flaps. Depending on tumor characteristics, adjuvant radiation therapy may be recommended.

Angiography will evaluate the arterial flow of the extremity and may be important in planning coverage of the wound, but it is not required to rule out a malignancy. MRI, CT scan, and x-ray studies can also be helpful as adjunctive diagnostic techniques to evaluate the extent of the tumor and presence of metastasis, but they are not used for diagnosis alone.

251
Q

An otherwise healthy 30-year-old man is evaluated because of left foot drop after posterior knee dislocation. The common peroneal nerve is explored at the level of the knee shortly after the time of injury and found to be in continuity. The patient does not return for follow-up examination within the next year. Eighteen months after the injury, the patient returns for follow-up examination and shows no improvement of the foot drop. Passive range of motion of the ankle is full. Which of the following transfers is most likely to correct this patient’s foot drop deformity?
A) Peroneus brevis tendon with graft to calcaneus bone
B) Peroneus longus tendon to calcaneus bone
C) Peroneus longus tendon to talus bone
D) Tibialis anterior nerve to tibialis posterior nerve
E) Tibialis posterior tendon to tibialis anterior tendon

A

E) Tibialis posterior tendon to tibialis anterior tendon

Common peroneal nerve injury is common after posterior knee dislocation, usually occurring at or near where the nerve crosses the fibula neck. After surgical exploration to confirm the peroneal nerve is intact, initial management consists of supportive care with an ankle brace to correct foot drop. Patients are observed clinically for recovery, often with serial electromyography and nerve conduction studies.

Tibialis posterior to anterior transfer will restore the patient’s ability to dorsiflex the ankle. The donor muscle is innervated by the tibial nerve, which is not commonly injured in a posterior knee dislocation. The tendon is dis-inserted from the tarsus and brought out through the medial leg.

It is transferred through the interosseous membrane to the tibialis anterior tendon.

By 18 months after injury, the motor end plates to the tibialis anterior have degenerated, making a nerve transfer no longer an option. Earlier after injury, a nerve transfer from the tibialis posterior to the tibialis anterior might be a feasible option.

Peroneus longus and brevis would also be paralyzed in the setting of a common peroneal nerve injury such as this patient’s. In certain situations, such as with some patients with leprosy, the deep peroneal nerve is not paralyzed. For these patients, the peroneus longus can be used as a tendon transfer, but it is normally used to provide toe extension in conjunction with a tibialis posterior transfer to provide ankle dorsiflexion.

252
Q
A 45-year-old man is brought to the emergency department after sustaining a fracture of the neck of the fibula after being struck by a baseball. Physical examination shows major nerve deficit. Which of the following actions will the patient be unable to perform?
A) Dorsiflex the foot
B) Extend the leg
C) Flex the leg
D) Invert the foot
E) Plantar flex the foot
A

A) Dorsiflex the foot

The most commonly injured nerve in the leg is the common peroneal nerve because of its superficial location as it courses around the neck of the fibula. The common peroneal nerve then divides into the superficial and deep branches. The superficial branch will evert the foot by innervating the lateral compartment of the leg, while the deep branch will dorsiflex the foot by innervating the anterior compartment. The superficial branch also provides sensation for the anterior and lateral sides of the leg and the majority of the dorsum of the foot and toes, including the medial side of the big toe. Paralysis of the common peroneal nerve would lead to foot drop and foot inversion, abnormal “steppage” gait, and loss of sensation.

253
Q
A 47-year-old woman is referred by orthopedic surgery for evaluation and discussion of soft-tissue reconstruction at the time of nonvascularized allograft reconstruction of recurrent Achilles tendon rupture. The patient has a history of congenital clubfoot and multiple previous Achilles tendon repairs. Physical examination shows atrophied skin and multiple longitudinal scars along both the medial and lateral distal posterior calf. Which of the following is the most appropriate management?
A) Cross-leg fasciocutaneous flap
B) Fasciocutaneous free flap
C) Reverse sural artery flap
D) Soleus muscle flap
E) Tissue expansion
A

B) Fasciocutaneous free flap

On the basis of the scenario described, fasciocutaneous free flap is the most appropriate management option.

The soleus muscle flap is appropriate for defects of the middle third of the leg but lacks adequate reach for soft-tissue coverage of the distal third of the leg.

Tissue expansion has been described for soft-tissue reconstruction of congenital talipes equinovarus but is usually reserved for children and in the setting of primary correction. When comparing tissue expansion in the limb versus non-limb sites, the incidence of complications associated with tissue expansion is significantly higher in the limb. Because a nonvascularized allograft is to be used, and the patient has a contracted and scarred soft-tissue envelope, tissue expansion would be associated with higher risk of expansion failure and complications when compared with free tissue transfer soft-tissue reconstruction.

Cross-leg flaps are rarely used because of the availability of free tissue transfer. This flap is more appropriate in children than elderly patients, in whom stiffness is a factor.

A reverse sural artery flap is not appropriate given the patient’s multiple past surgeries and local scars.

254
Q
A 29-year-old man undergoes open reduction and internal fixation of an open fracture of the proximal right tibia. There is no tissue loss, and there is little wound contamination. The wound is closed with 2-cm raised flaps. Reconstruction of the popliteal artery is required. Which of the following Gustilo fracture classifications is most likely in this patient?
A) I
B) II
C) IIIA
D) IIIB
E) IIIC
A

E) IIIC

Gustilo initially classified long-bone fractures into three types in order to establish a treatment algorithm. Essentially, this classification subdivided fractures according to the energy of the initial trauma that resulted in significant soft-tissue injury, periosteal stripping, and fracture comminution in the worst subtype. Debridement, antibiotics, and primary or delayed wound closure were advocated dependent on fracture severity. Type III fractures were subsequently subdivided into A, B, and C subtypes. Subtypes were stratified according to potential for complications such as infection, osteomyelitis, non-union, and amputation rates. Type IIIC had open fracture with arterial injury requiring repair (the case in this patient, even though there appears to be adequate soft-tissue coverage).

Although fracture fixation methods have substantially improved since the original publications of Gustilo, the ability to transport bone into segmental traumatic defects has also since developed, and free flaps have extended our ability to cover large wounds. This classification system has stood the test of time and still forms the basis of prognosticating and determining the optimum treatment algorithm.

The Gustilo grading scale:
Type Findings
I Clean wound bed, simple/minimally comminuted bone injury, wound 1 cm
IIIA Wound highly contaminated, severe comminution, wound 1 to 10 cm
IIIB Wound highly contaminated, severe comminution, wound >10 cm
IIIC Major vascular injury requiring repair for limb salvage

255
Q

A 57-year-old man comes to the office 4 weeks after undergoing a free osseocutaneous fibula flap. He says he has pain with walking. A photograph is shown. X-ray studies show 6 cm of fibular bone remains proximally and distally. Sensation of the right foot shows no abnormalities; pain is noted on plantar flexion. Which of the following is the most appropriate next step in management?
A) Cast immobilization of the lower extremity (above the knee)
B) Cast immobilization of the lower extremity (below the knee)
C) Operative exploration and bone grafting
D) Operative exploration and nerve grafting
E) Reassurance that the pain is self-limiting

A

E) Reassurance that the pain is self-limiting

Vascularized bone flap is typically needed for defects >6 cm regardless of location in the body. The fibula is a common donor for vascularized bone. Understanding the postoperative course and complications is needed both in terms of discussions with the patient preoperatively and management of the patient’s condition after surgery. Common sequelae of fibula harvest include pain in the leg (especially when walking). Four weeks is relatively early in the postoperative course and reassurance should be given.

Risks of fibula harvest include damage to the peroneal nerve (increased when

256
Q

A 32-year-old woman comes to the office because the toes of the right foot “drag” when she walks. She underwent vein stripping of the right leg and ligation of the lesser saphenous vein 4 weeks ago. Physical examination shows absent dorsiflexion and eversion of the ankle. Electromyography findings show:

Muscle Group Recruitment
Biceps femoris +
Tibialis posterior + 
Peroneus longus -
Tibialis anterior -
EHL -

Which of the following is the most likely site of nerve injury in this patient?

A) Common peroneal nerve at the knee
B) Superficial peroneal nerve at the knee
C) Sural nerve at the knee
D) Tibial nerve at the knee
E) Tibial nerve at the mid calf
A

A) Common peroneal nerve at the knee

The most likely site of injury would be the common peroneal nerve at the knee. Injuries to the common peroneal nerve are well documented in both traumatic (knee dislocation) and iatrogenic settings. Patients have footdrop and numbness over the first dorsal web space of the foot. History and clinical examination are the mainstays for diagnosis, but electromyography can be helpful in less-clear circumstances. The absence of recruitment of the lateral compartment muscles (peroneals) and the anterior compartment muscles (tibialis anterior, extensor hallucis longus) strongly suggest common peroneal involvement. The presence of recruitment of the biceps femoris and the tibialis posterior rules out tibial nerve involvement. An isolated superficial peroneal nerve injury would spare the anterior compartment muscles.

The sural nerve is a sensory nerve and provides no motor function.

257
Q

A 24-year-old man is brought to the emergency department 2 hours after sustaining injuries to the left lower extremity when he was hit by a motor vehicle. Physical examination shows avulsion of the soft tissue of the posterior thigh. A fracture of the femur is stabilized by an intramedullary rod; the sciatic nerve is noted to be intact but ecchymotic at the level of the mid posterior thigh. Soft tissue is available for coverage. Which of the following is the most appropriate management?
A) Acute resection of the ecchymotic nerve and repair with a nerve graft
B) Delayed resection of the ecchymotic nerve at 10 days and repair with a nerve graft
C) Electromyography after 3 weeks and repair with a nerve graft if fibrillations occur
D) Serial electrodiagnostic studies after 3 weeks and again after 3 months with repair if no improvement

A

D) Serial electrodiagnostic studies after 3 weeks and again after 3 months with repair if no improvement

In cases in which the nerve has undergone a significant crush component, it is important to get a sense of nerve viability and recovery. The nerve conduction study at 3 weeks largely serves as a baseline study as it rarely provides more information than physical examination other than the presence of fibrillations, which indicates at least some axonal loss. The nerve conduction study at 3 weeks is not a reliable indicator of possible nerve recovery; therefore, resection and reconstruction are not advisable at this time point. The 3-month nerve conduction study is able to pick up subtle signs of recovery that may not be evident on physical examination. If at the 3-month mark there are no signs of recovery on physical examination or nerve conduction study, repair is indicated. Acute resection of a possibly viable nerve is not indicated. If the nerve were noted to be transected at the time of initial exploratory surgery, the viability of the nerve ends would not be stable until 7 to 10 days post trauma, making this a good time for definitive repair. In the scenario described, the nerve is in continuity and viability cannot be ascertained intraoperatively at 10 days; therefore, resection and reconstruction are not indicated.

258
Q

A 35-year-old man is brought to the emergency department 2 hours after sustaining a severe crush injury to the right distal thigh in a motor vehicle collision. Physical examination shows an open fracture of the femur; the leg and foot are pale and cool. There are no palpable popliteal, dorsalis pedis, or posterior tibialis pulses. Closed reduction does not restore perfusion. Which of the following is the most appropriate next step?
A) Arterial repair with a polytetrafluoroethylene graft
B) Arterial repair with reverse saphenous vein graft
C) CT angiography
D) Intramedullary fixation of the femoral fracture
E) Placement of a temporary vascular shunt

A

E) Placement of a temporary vascular shunt

Gustilo Type IIIC fractures involve arterial injury requiring repair irrespective of the degree of soft tissue and often represent significant limb-threatening injuries. Early recognition and management of lower extremity vascular injury is crucial to limb salvage. CT angiography is of little benefit in the presence of hard signs of vascular injury and can delay operative intervention as well as increase limb ischemia time.

The combination of vascular and orthopedic injuries requiring repair is rare, with a reported incidence as low as 1.5%. Data exist from both wartime and civilian groups evaluating the sequence of management of such injuries. The recommended algorithm suggests improved ischemia times and favorable limb salvage rates with temporary vascular repair, using shunts as the initial adjunct to restore perfusion followed by debridement and fracture fixation.

Definitive vascular repair should follow debridement and fracture fixation. Both synthetic polytetrafluoroethylene and autologous (reverse saphenous vein) interposition grafts are reported to be used in traumatic reconstruction, although autologous tissue is often preferred in the setting of gross contamination.

259
Q
A 25-year-old man is brought to the emergency department after he sustained a knife wound to the right lower extremity. Examination shows numbness of the lateral aspect of the leg and weakness in plantar flexion and eversion of the foot. Which of the following nerves was most likely injured in this patient?
A) Femoral
B) Obturator
C) Peroneal
D) Sural
E) Tibial
A

C) Peroneal

The patient appears to demonstrate symptoms of a superficial peroneal nerve injury. The superficial peroneal nerve arises from the common peroneal nerve at the fibular neck. It supplies the lateral compartment of the leg, giving motor branches to peroneus longus and brevis, as well as sensory contribution to the lateral aspect of the leg. Injury to the superficial peroneal nerve results in anesthesia of the lateral aspect of the leg and weakness in eversion and plantar flexion of the foot.

The deep peroneal nerve arises from the common peroneal nerve at the fibular neck. It travels in the anterior compartment of the leg and gives branches to the tibialis anterior, extensor hallucis longus, and extensor digitorum longus and brevis, as well as peroneus tertius. The sensory distribution is in the area of the first web space. Injury to the deep peroneal nerve causes weakness in dorsiflexion of the foot.

The femoral nerve innervates muscles of the anterior thigh, including the quadriceps group, iliacus, and sartorius. Injury to the femoral nerve results in weakness of leg extension.

The obturator nerve provides innervation to the medial thigh muscles (adductor group), including adductor brevis, longus, and magnus, as well as the gracilis and obturator externus. The cutaneous branch provides sensation of the medial thigh. Injury to the obturator nerve results in weakness in thigh adduction and sensory deficits in the medial thigh.

The sural nerve travels on the posterior aspect of the leg between the lateral malleolus and calcaneus. It provides sensation to the lateral aspect of the foot and does not have a motor component. It is commonly sampled in nerve biopsy and used as a source of nerve graft.

Injury or sacrifice of the sural nerve would result in numbness of the lateral foot.

The tibial nerve is a branch of the sciatic nerve. It travels through the popliteal fossa and gives off branches to gastrocnemius, soleus, plantaris, and popliteus muscles. The tibial nerve travels in proximity to the posterior tibial artery. In the leg, it gives off branches to the flexor digitorum longus, tibialis posterior, and flexor hallucis longus. Distally in the foot, it branches to give rise to the medial and lateral plantar nerves, which provide sensation to the plantar surface of the foot. Injury to the tibial nerve results in deficits of plantar flexion, as well as anesthesia to the plantar surface of the foot.

260
Q
A 24-year-old man comes to the office 3 months after closed reduction of a right knee dislocation. His knee is stable, but he still depends on an ankle/foot orthosis for ambulation. Physical examination shows decreased light-touch sensation along the dorsolateral aspect of the foot. Ankle eversion is absent. Sensation and motor function are otherwise intact. Nerve conduction testing is most likely to demonstrate a block in which of the following nerves?
A) Common peroneal
B) Lateral plantar
C) Medial plantar
D) Posterior tibial
E) Superficial peroneal
A

E) Superficial peroneal

Common peroneal nerve injuries involving motor function loss have been reported in up to 50% of knee dislocations. If isolated sensory disturbances are also included, the incidence of nerve injury approaches 75%. If no recovery is noted by 3 to 6 months following injury, then surgical treatment is warranted. Physical exam primarily determines the nerve to be explored, neurolysed, and possibly grafted, but nerve conduction studies can be useful pre- and intraoperatively.

The common peroneal nerve divides into three branches at the knee, an articular branch that innervates the joint capsule and lateral collateral ligament of the knee, the superficial, and deep branches. The superficial branch innervates the muscles of the lateral compartment of the leg and provides sensation to the lateral calf and dorsal foot. The deep branch innervates the anterior compartment and provides sensation to the first web space of the foot. The scenario given above is most consistent with compromise of the superficial peroneal nerve. If dorsiflexion of the ankle and toe extension had also been lost, then common peroneal nerve injury would have been suggested.

The posterior tibial nerve proper innervates the muscles of the posterior calf, mediating ankle plantar flexion and toe flexion. The medial and lateral plantar nerves are terminal branches of the posterior tibial nerve. They provide motor innervation to the deep plantar muscles of the foot and sensation to the plantar surface of the foot.

261
Q
A 25-year-old construction worker has a 4-cm-diameter posterior calcaneal ulcer with exposed bone on removal of a short-leg cast applied 6 weeks ago for an ankle fracture. Coverage with a lateral calcaneal artery flap is planned. The lateral calcaneal artery is usually the terminal branch of which of the following arteries?
A) Anterior tibial
B) Dorsalis pedis
C) Lateral malleolar
D) Peroneal
A

D) Peroneal

Anatomic dissections by Drs. Grabb and Argenta found that the lateral calcaneal artery is usually the terminal branch of the peroneal artery but occasionally may arise from the posterior tibial artery. The branches of the peroneal include the nutrient artery which supplies the fibula, the perforating branch which gives branches to the tarsus, the communicating branch and the lateral calcaneal.

262
Q
During harvest of a plantaris tendon graft, which of the following structures is at greatest risk for injury?
A) Dorsalis pedis artery
B) Extensor hallucis brevis muscle
C) Medial plantar artery
D) Sural nerve
E) Tibial nerve
A

E) Tibial nerve

When multiple tendon grafts are needed or when it is necessary to harvest grafts long enough to reach from the forearm to the fingertip, lower extremity tendon graft harvest is necessary. The plantaris tendon is a good source of tendon graft and is present in about 80% of limbs.

The graft is harvested through a vertical incision just anterior to the medial aspect of the Achilles tendon. Then the graft is followed proximally using either a tendon stripper or with further incisions. As such, dissection of the plantaris tendon begins behind the medial malleolus and close to the tibial nerve. The sural nerve lies about the lateral malleolus and thus is not likely to be inadvertently injured during plantaris harvest. The medial plantar artery is on the sole of the foot and would be distal to the field of dissection. The extensor hallucis brevis muscle is a small muscle that lies over the dorsum of the foot and thus, like the dorsalis pedis artery, would not be injured in the dissection.

263
Q
The primary blood supply to a free anterolateral thigh fasciocutaneous flap arises from vessels that perforate which of the following muscles?
A) Gluteus maximus
B) Rectus femoris
C) Sartorius
D) Tensor fascia lata
E) Vastus lateralis
A

E) Vastus lateralis

The anterolateral thigh (ALT) flap has proven to be one of the most versatile free tissue transfers in reconstructive surgery. Based on perforators from the descending branch of the lateral circumflex femoral artery that traverse the vastus lateralis (VL) (80%) or the septum between the rectus femoris and VL (18-20%), this flap can be fashioned as large as 10 cm wide by 25 cm long. Occasionally, no large perforator will be identified during dissection; in this circumstance, the flap may be carried on multiple perforators along with the vastus lateralis muscle.

Branches of the lateral circumflex femoral artery also supply the sartorius (partial, as the supply is segmental), rectus femoris (descending branch), and tensor fascia lata (ascending branch). Vascular supply to the gluteus maximus arises from the superior and inferior gluteal arteries.

264
Q
A 30-year-old man is scheduled to undergo great toe-to-thumb transfer 7 months after traumatic amputation of the dominant thumb. During dissection of the toe, the first dorsal metatarsal artery is most likely to be found branching from which of the following vessels?
A) Deep plantar
B) Dorsalis pedis
C) Plantar arterial arch
D) Posterior tibial
E) Proper digital
A

B) Dorsalis pedis

The origin and course of the first dorsal metatarsal artery (FDMA) are key to dissecting the first or second toe and the variety of available toe flaps. This anatomy is quite variable. In two thirds of cases, this artery emanates from the dorsalis pedis artery as its distal continuation. This course can then be superficial, within, or deep to the interosseous muscle. However, in one third of patients, the metatarsal artery may arise from the deep plantar artery that communicates with the plantar arch or actually from the plantar arch itself, in which case the FDMA may be vestigial. In the latter two situations, the metatarsal artery passes plantar to the deep transverse metatarsal ligament. The proper digital arteries are the distal continuations of the FDMA. The posterior tibial artery runs longitudinally in a superficial plane to the forefoot on the plantar surface. It is the larger lateral plantar artery that travels deeply to become the plantar arterial arch.

265
Q

An otherwise healthy 47-year-old man is transferred to the hospital because of an infection of the leg. He sustained the initial injury in a fall 6 weeks ago that was treated with internal fixation. The infection is now under control, and the internal hardware has been removed. Examination shows a 9-cm bony defect of the lower extremity. Neurovascular status of the foot is normal. Angiography of both lower extremities shows no abnormalities. A photograph and x-ray study are shown. Which of the following considerations favors vascularized bone grafting in this patient?
A) Length of time since the initial injury
B) Mechanism of the injury
C) Method of injury stabilization
D) Patient age
E) Size of the bony defect

A

E) Size of the bony defect

The injury described is a Gustilo IIIB lower extremity wound complicated by infection. The sequence of reconstruction is often bony stabilization and debridement until bacterial balance. Bony deficits can be reconstructed in a variety of ways, including non-vascularized grafts, vascularized grafts (free of pedicle), and bone transport.

Generally, for defects greater than 6 to 8 cm, vascularized bone grafting is indicated.

Other indications for vascularized bone grafting are the presence of infection and prior failure of conventional (non-vascularized) grafting.

Age of the patient is not a contraindication to reconstruction per se, as long as he or she is medically stable to undergo a prolonged operation.

Gustilo Classification
I: open fracture; clean; wound less than 1 cm
II: open fracture; wound greater than 1 cm
IIIA: open fracture; extensive soft-tissue injury but adequate tissue for coverage
IIIB: open fracture; extensive soft-tissue injury but inadequate tissue for coverage
IIIC: any of the above with a vascular (arterial) injury

266
Q
A 37-year-old man comes to the office because of wound breakdown 2 weeks after he sustained a calcaneal fracture. Orthopaedic stabilization was performed in the emergency department at the time of the injury. Examination today shows a 3 × 4-cm wound over the lateral calcaneus. Coverage with a propeller fasciocutaneous flap from the lateral leg is planned. Which of the following blood vessels supplies the perforators of this flap?
A) Anterior tibial artery
B) Lateral plantar artery
C) Lateral sural artery
D) Peroneal artery
E) Posterior tibial artery
A

D) Peroneal artery

Propeller flaps are a useful method of lower extremity reconstruction. These flaps were initially popularized by Teo and have been utilized for a variety of defects in the lower leg and foot. The propeller flap is based on perforating blood vessels from the peroneal artery to reconstruct lateral defects and perforators from the posterior tibial artery to reconstruct defects on the medial aspect of the leg wound. The propeller flaps can often replace the need for a sural artery, neurocutaneous artery flap or a free tissue transfer. Laterally based plantar flaps may be used to cover small defects on the weight-bearing surface of the foot but are not reliable for larger or lateral defects. The posterior and anterior tibial arteries do not have perforator in the desired location for a distally based propeller flap. The lateral sural vessels are too proximal for heel coverage.

267
Q
A 55-year-old man is evaluated in the emergency department for foot salvage after he was involved in a motorcycle collision. Flow is restored after 6 hours from the time of injury. On examination, the foot is cold, and no plantar sensation is noted. The posterior tibial nerve is disrupted. A temporary external fixator is placed. A photograph and an x-ray study are shown. Which of the following is the most appropriate classification of this injury and recommendation for management?
A) Gustilo type IIIB; amputation
B) Gustilo type IIIB; reconstruction
C) Gustilo type IIIC; amputation
D) Gustilo type IIIC; reconstruction
A

C) Gustilo type IIIC; amputation

The patient described has a Gustilo IIIC injury. Based on the best available data, he should undergo amputation.

Ultimately, the choice to reconstruct versus amputate is a gestalt of the situation and the patient, as well as the capabilities of the hospital and the care team. In this case, the factors influencing the decision would be the warm ischemia time of 6 hours and severed posterior tibial nerve, as well as the extensive soft-tissue injury.

Some of the newer data suggests that absence of plantar sensation is no longer criteria for amputation in and of itself. However, an anatomically disrupted nerve in an adult strongly favors amputation.

In addition, there is evidence supporting the notion that limb salvage might involve less cost in the long term versus reconstruction.

Gustilo Classification
I: open fracture; clean; wound less than 1 cm
II: open fracture; wound greater than 1 cm
IIIA: open fracture; extensive soft-tissue injury but adequate tissue for coverage
IIIB: open fracture; extensive soft-tissue injury but inadequate tissue for coverage
IIIC: any of the above with a vascular (arterial) injury

268
Q
A 48-year-old man comes to the office because of drainage from the distal aspect of the wound 2 weeks after he underwent soft-tissue coverage with a free latissimus dorsi muscle flap for a degloving injury of the left lower extremity sustained in a motorcycle collision. Examination in the emergency department showed a Gustilo type IIIB tibia/fibula fracture and 6 cm of tibial loss. Debridement of the bone and stabilization with an external fixator were performed at the time of the injury. The patient underwent soft-tissue coverage 8 days after the injury. Examination today shows purulent drainage at the lower portion of the flap. Which of the following is the most likely cause of this complication?
A) Age of patient
B) Delay in soft-tissue coverage
C) Distal flap necrosis
D) Inadequate debridement
A

D) Inadequate debridement

Posttraumatic reconstruction of the lower extremities with significant soft-tissue defects that expose bone, joints and tendons generally require free tissue transfer. The Gustilo-Anderson fracture classification system is widely used to describe the injury when associated with a long bone fracture. Gustilo type IIIB fractures are associated with periosteal stripping and exposure of bone with contamination. In 1986, Godina emphasized coverage of these soft-tissue defects within the first 72 hours of injury. Given the nature of these poly-trauma injuries, coverage of these wounds is not always possible in the given time frame. Staged debridement and negative pressure therapy have become common in the management of these injuries. Inadequate bone or soft-tissue debridement prior to reconstruction is a common cause of failure. The wound has to be free of all contaminants prior to reconstruction; thus, it is unlikely that an 8-day delay is the cause of breakdown. Distal flap necrosis would show more superficial breakdown and, in this case, is not the cause for a deep space infection. The patient’s age does not have any bearing on this complication.

269
Q

Which of the following is the most likely cause of congenital constriction band syndrome of the lower extremity?
A) Defect in the zone of polarizing activity
B) Deletion of the gene responsible for the apical ectodermal ridge
C) Early amnion rupture
D) Exposure to retinoic acid
E) Use of thalidomide during pregnancy

A

C) Early amnion rupture

Early amnion rupture with subsequent entanglement of fetal parts (mostly limbs and appendages) by amniotic strands is the primary theory of pathogenesis.

A wide spectrum of clinical deformities is encountered and range from simple ring constrictions to major visceral defects. Lower extremity limb malformations are extremely common and consist of asymmetric digital ring constrictions, distal atrophy, congenital intrauterine amputations, acrosyndactyly, lymphedema, and clubfoot.

Amniotic band syndrome is not a rare anomaly, as first described more than 150 years ago, and appears to be rising. Once believed to have an incidence of 1:100,000, recent literature supports the incidence today as 1:1200 to 1:5000 births.

No distinct sex predilection has been determined. Nearly 60% of the cases documented have some sort of abnormal gestation history. Prenatal risk factors associated with amniotic band syndrome include prematurity (less than 37 weeks), low birth weight (less than 2500 g), maternal illness during pregnancy, maternal drug exposure, and maternal hemorrhage. Attempted abortion in the first trimester is also a highly associated finding. Family history seldom shows any direct inheritance pattern since the syndrome occurs in no particular association with known genetic or chromosomal disorders.

The zone of polarizing activity signals the developing limb bud towards anterior/posterior polarity and does not result in truncation.

Proximodistal limb growth is the result of the apical ectodermal ridge. Deletion of the gene responsible for the apical ectodermal ridge will result in shortening of the limb but not in congenital constriction band syndrome. Experiments in which the apical ectodermal ridge has been removed show truncated limb growth. Scientific evidence supports that the pattern of limb anomalies in the offspring of mothers exposed to exogenous retinoids, such as retinoic acid, causes interference with apical ectodermal ridge function, resulting in similar deformities.

Thalidomide results in various limb deformities, including phocomelia, dysmelia, amelia, and bone hypoplasticity, with as little as a single dose of thalidomide during gestation. It does not, however, result in constriction bands. It is postulated that thalidomide-associated malformations are the result of the drug’s interference with vasculogensis.

270
Q

A 17-year-old boy sustains an avulsion injury to the anterior ankle with exposed tendon in a motorcycle collision. The wound is evaluated and reconstruction with a reverse sural artery flap is planned. During elevation of the flap, which of the following is most likely to compromise flap viability?
A ) Injury to the lesser saphenous vein
B ) Injury to the median superficial sural artery
C ) Injury to the sural nerve
D ) Ligation of the gastrocnemius muscle perforators
E ) Ligation of the peroneal artery perforators

A

E ) Ligation of the peroneal artery perforators

The predominant blood supply to the reverse sural artery flap relies on perforators from the peroneal artery, which primarily originate at a point 5 cm proximal to the lateral malleolus.

The median superficial sural artery and perforators from the gastrocnemius are involved in anterograde blood supply to the flap. Thus, these vessels are typically ligated in the reverse form of the flap.

Likewise, injury to the lesser saphenous vein would not cause significant problems with venous outflow. In dissections and venous flow studies, small concomitant veins were found along both sides of the lesser saphenous vein and were considered to be venae comitantes of accompanying arteries of the vein. These accompanying veins played a role in bypassing the valves of the lesser saphenous vein.

The sural artery flap can be harvested as a neural-veno-adipofascial flap. In this context, the sural nerve can used to provide sensation to the flap. This nerve does not provide primary blood supply to the flap, and injury would not compromise flap viability.

271
Q
A 55-year-old man comes for evaluation of a nonhealing foot ulcer. He has a 10-year history of type 1 diabetes mellitus. Physical examination shows a deep, 4-cm plantar ulcer at the great toe metatarsal head. Which of the following is the most likely cause of this ulcer?
A ) Atherosclerosis
B ) Lymphatic obstruction
C ) Peripheral neuropathy
D ) Poor hygiene
E ) Venous stasis
A

C ) Peripheral neuropathy

Sensory neuropathy leads to loss of protective sensations. This then leads to neuropathic imbalance with loss of coordination which increases mechanical stresses, unperceived trauma, Charcot foot, and ulcers. Sympathetic neuropathy leads to warm, dry feet which are prone to skin breakdowns. Patients with diabetes who also have peripheral vascular disease are more prone to ischemic ulcers. Patients with diabetic foot ulcers have not been associated with large-vessel atherosclerosis.

272
Q
A 25-year-old man undergoes open reduction and internal fixation of a Gustilo Type IIIB tibia-fibula fracture. Coverage of the resultant soft-tissue defect with a free flap is planned. Which of the following is the ideal length of time after the injury that this procedure should be performed?
A ) 3 Days
B ) 10 Days
C ) 21 Days
D ) 60 Days
E ) 90 Days
A

A ) 3 Days

Godina defined the timing of posttraumatic microsurgical lower extremity reconstruction in 1986 with recommendations for flap coverage of Gustilo Type IIIB/C fractures within 72 hours of injury. Godina’s study showed the highest risk of infection and flap loss in the delayed period (72 hours to 90 days). Subsequent authors have also cited lower rates of flap loss and infection when repair was performed.

Many authors have reported good flap survival when surgery is done within 3 days and 3 months after injury, but it is accepted that this delayed time frame necessitates more meticulous dissection and going well outside the zone of injury to a patent venous system to decrease the incidence of free flap loss.

273
Q

A 66-year-old man comes to the office with a 2-year history of progressive ulceration at the right medial malleolus. He says he has marked calf pain when he is active, and that it has worsened progressively over time. Medical history includes cardiovascular disease and a 40-year history of smoking. Physical examination shows that the wound margins are well demarcated with a yellow fibrinous wound base. Which of the following is the most appropriate management of this patient?s condition?
A ) Biopsy of the wound edge
B ) Debridement and dressing changes
C ) Debridement and local flap coverage
D ) Revascularization by vascular surgery
E ) Use of topical debriding agents

A

D ) Revascularization by vascular surgery

Arterial ulcers most commonly occur in male patients with atherosclerosis, usually in association with cardiovascular disease, diabetes, hypertension, or smoking. Patients typically have symptoms of claudication and rest pain. The leg pain improves when the leg is dependent and is exacerbated when the leg is elevated. The base of the arterial ulcer generally does not bleed and has a ?punched-out? appearance. Ulcers associated with an ankle brachial index of less than 0.45 (normal range 0.90 to 1.30) generally do not heal without revascularization. Aggressive debridement would create a larger wound without the potential to heal. Continued wound care would provide no long-term benefit. The clinical picture described is not consistent with malignant degeneration of the wound.

274
Q

An otherwise healthy 25-year-old man is evaluated because of footdrop and paresthesia 4 weeks after undergoing open reduction and internal fixation of a fracture of the proximal tibia. Electrodiagnostic testing shows no conduction of the nerve. Which of the following is the most appropriate next step in management?
A ) Ankle arthrodesis
B ) Peroneal nerve decompression
C ) Posterior tibial nerve decompression
D ) Posterior tibial tendon transfer
E ) Repeat electrodiagnostic testing in 4 weeks

A

E ) Repeat electrodiagnostic testing in 4 weeks

Peroneal nerve palsy may lead to severe disability with footdrop and paresthesia. Traumatic peroneal palsy may result from supracondylar femoral fracture, knee dislocation, and proximal tibial fracture involving both motor and sensory changes. On examination, the motor deficit manifests as a footdrop, and the sensory deficit results in a loss of sensation along the dorsum of the foot. Unlike the loss of sensation associated with the peroneal nerve, the posterior tibial nerve is clinically morbid, as it is on the plantar aspect of the foot. The chronic injury may result in wounding of the plantar aspect of the foot, which may ultimately result in amputation.

Electrodiagnostic testing should be performed within 1 month of the injury to confirm sensory and motor deficits. In patients with partial nerve palsy, greater than 80% will recover completely and can be expected to recover with observation alone. A patient with complete palsy will have a much worse prognosis, with only 40% of patients with a functional recovery. If there is no neurologic improvement after 2 to 3 months, operative decompression should be performed. If the nerve is noted to be in continuity, and nerve action potentials are transmitted across a lesion, then there may be continuity of the nerve, and neurolysis is performed. If there is a transected nerve, a direct repair or nerve grafting may be performed.

When neurolysis and nerve grafting do not provide satisfactory results, the remaining surgical options are salvage procedures such as tendon transfer or arthrodesis. Posterior tibial tendon transfer is the most commonly used tendon transfer for this injury.

275
Q

A 58-year-old man comes to the office for follow-up 3 weeks after undergoing mandibular reconstruction with a fibular free flap. The patient reports numbness of the operated extremity and says he has difficulty walking. Which of the following sensory and motor deficits is most likely in this patient?

Absence of sensation vs Weakness
A) First web space of the foot vs EHL and TA
B) Medial aspect of the calf vs Soleus and EDBrev
C) Medial malleolus and heel vs Soleus and TA
D) Third web space of the foot vs Soleus and EDBrev
E) Upper lateral calf vs FHL and EDBrev

A

A) First web space of the foot vs EHL and TA

Both the first web space of the foot and the extensor hallucis longus and tibialis anterior muscles are innervated by branches of the common peroneal nerve. The finding of absence of sensation over the medial aspect of the calf and weakness of the soleus and extensor digitorum brevis muscles is eliminated by numbness of the medial calf, which is innervated by the saphenous nerve, a branch of the femoral nerve. Absence of sensation over the medial malleolus and heel with weakness of the soleus and tibialis anterior muscles is eliminated by numbness of the medial malleolus, which is also innervated by the saphenous nerve. In addition, the soleus is innervated by the tibial nerve. The finding of absence of sensation in the third web space of the foot and weakness of the soleus and extensor digitorum brevis muscles is eliminated by the soleus muscle. The finding of absence of sensation over the upper lateral calf and weakness of the flexor hallucis longus and extensor digitorum brevis muscles is eliminated by the flexor hallucis longus, which is innervated by the tibial nerve.

276
Q
A 46-year-old man is brought to the emergency department after sustaining an open comminuted fracture of the left tibia during a motor vehicle collision. Physical examination shows a wound greater than 10 cm with marked periosteal stripping near the mid shaft. Which of the following is the appropriate Gustilo classification of this patient's injury?
A ) I
B ) II
C ) IIIA
D ) IIIB
E ) IIIC
A

D ) IIIB

The Gustilo classification of open fractures is based on the severity of soft-tissue injury and the presence or absence of concurrent vascular injury. A higher grade implies a more severe injury, and correlates with a greater incidence of complications and need for amputation.

The patient in this item has a wound greater than 10 cm and a heavily comminuted tibia fracture. There is no mention of vascular injury. He has a Gustilo type IIIB injury.

277
Q
A 23-year-old man is brought to the emergency department 4 hours after sustaining an open tibia and fibula fracture to the middle third of the leg in a motorcycle collision. A proximally based soleus flap is chosen to reconstruct the defect. Which of the following arteries is the most likely blood supply of this flap?
A ) Anterior tibial
B ) Inferior geniculate
C ) Peroneal
D ) Profunda femoris
E ) Superior geniculate
A

C ) Peroneal

The soleus muscle is a broad, powerful muscle of the posterior calf which, along with the gastrocnemius muscle, forms the triceps surae. The soleus takes its origin from the upper third of the fibula and medial border of the tibia and inserts into the calcaneus with the gastrocnemius muscle by way of the Achilles tendon. The blood supply of the soleus muscle is from the peroneal artery proximally and the posterior tibial artery distally. This soleus muscle flap is used as a pedicle muscle flap for coverage of defects of the middle third of the lower leg.

278
Q
Which of the following is the most likely indication to consider replantation of a lower extremity?
A ) Bilateral sharp injury
B ) Crush mechanism of injury
C ) Ischemia time over 8 hours
D ) Multilevel injury
E ) Patient over age 50 years
A

A ) Bilateral sharp injury

The indications for lower extremity replantation are limited. The best candidate for a lower extremity replantation would be a young patient who has had both legs amputated with a clean mechanism. Contraindications for lower extremity replantation include crush mechanism of injury, ischemia time over 8 hours, multiple-level injury, poor baseline health, and a patient of advanced age.